[Ответить в тред] Ответить в тред

16/08/16 - Запущен Двач Трекер
01/08/16 - Вернули возможность создавать юзердоски
09/07/16 - Новое API для капчи - внимание разработчикам приложений



Новые доски: /obr/ - Offline Battle Rap • /hv/ - Халява в интернете • /2d/ - Аниме/Беседка • /wwe/ - WorldWide Wrestling Universe • /ch/ - Чатики и конфочки • Создай свою

[Назад][Обновить тред][Вниз][Каталог] [ Автообновление ] 536 | 44 | 145
Назад Вниз Каталог Обновить

МАТЕМАТИКА ДЛЯ НАЧИНАЮЩИХ, ТРЕД 6, ПОМОЩЬ НОВИЧКАМ Аноним 14/05/16 Суб 23:33:26  362618  
14632580065670.jpg (15Кб, 275x326)
14632580065681.jpg (8Кб, 262x326)
14632580065682.jpg (19Кб, 228x271)
14632580065683.jpg (179Кб, 818x1000)
ДЛЯ САМЫХ МАЛЕНЬКИХ:

Общие курсы
М. И. Сканави: "Элементарная математика".

Алгебра
И. М. Гельфанд, А. Шень: “Алгебра”. Весь курс школьной алгебры по 9 класс.
С. Б. Гашков: “Современная элементарная алгебра”.
Ю. М. Алимов, М. В. Колягин: "Алгебра и начала анализа".

Геометрия
Г. С. М. Коксетер: “Введение в геометрию“. Годная книга для уровня "продвинутый школьник".
А. Д. Александров, А. Л. Вернер, В. И. Рыжик: “Геометрия”. Учебник для 10-11 классов. Базовый и углубленный уровни.
Я.П. Понарин: “Элементарная геометрия” в двух томах. Собственно, первый том - это планиметрия, а второй том - это стереометрия.
А. Ю. Калинин, Д. А. Терешин: “Геометрия”, 10-11 классы. Годный учебник.

Тригонометрия
И. М. Гельфанд, С.М. Львовский, А. Л. Тоом: “Тригонометрия”. Название говорит само за себя. Много геометрических и физических интерпретаций + комплексные числа, как бонус.

Для поступающих в ВУЗ
В. В. Ткачук: “Математика - абитуриенту”. Один из лучших учебников для поступающих в ВУЗы.
Г. Н. Яковлев: “Пособие по математике для поступающих в ВУЗы”.


БАЗОВЫЕ КУРСЫ ДЛЯ СТУДЕНТОВ:

Теория доказательств
G. Chartrand, A. D. Polimeni: "Mathematical Proofs: A Transition to Advanced Mathematics". Очень хороший учебник, не только по основам, но и с забегом в различные области математики (в том числе и топологию с некоторыми разделами алгебры). На либгене есть третье издание и решебник для второго (в третьем больше задач, для недостающих в решебнике нечетных номеров есть ответы в конце книги).

Алгебра
Э. Б. Винберг: “Курс алгебры”. Пожалуй, лучший из известных учебников, соперничать с которым может разве что "Введение в алгебру" Кострикина.
А. И. Кострикин: “Введение в алгебру“. Пожалуй, лучший из известных учебников, соперничать с которым может разве что "Курс алгебры" Винберга.
В. А. Ильин, Э. Г. Позняк: “Линейная алгебра“. Один из классических и самых популярных курсов линейной алгебры.
Д. В. Беклемишев: “Курс аналитической геометрии и линейной алгебры“.
P. Grillet: "Abstract algebra".
J. Rotman: "Advanced modern algebra". Ротман сильно разжевывает. Задачи слишком простые для уровня учебника.
M. Artin: "Algebra". Американский Винберг. Группы Ли, упор на геометрию (классические линейные группы это все). Задачи неудачные.
I. N. Herstein: “Topics in Algebra“. Прекрасные задачи, отбор материала очень устарел, почти что Ван дер Варден.
P. Aluffi: "Algebra, Chapter 0". Если ты в состоянии ее осилить, бери и забывай про остальные книжки из списка.

Математический анализ
T. Tao: “Real analysis“. Один из самых популярных курсов математического анализа на английском языке.
Р. Курант: "Курс дифференциального и интегрального исчисления". Идеален с точки зрения первого знакомства с теорией, но имеет достаточно сложные упражнения.
Г. М. Фихтенгольц: "Курс дифференциального и интегрального исчисления". Хорош как повторительный курс.
Г. Г. Харди, Д. Е. Литтлвуд, Г. Пойа: "Неравенства".
Н. Н. Лебедев: "Специальные функции и их приложения".
Г. П. Толстов: “Ряды Фурье“.

Геометрия
A. Ostermann, G. Wanner: "Geometry by its history".
R. Vakil: "Foundations of algebraic geometry".

Дифференциальные уравнения
С. Фарлоу: “Уравнения с частными производными для научных работников и инженеров“.

Вариационное исчисление
И. М. Гельфанд, С. В. Фомин: " Вариационное исчисление".

Топология
V. Runde: "A taste of topology".
П. С. Александров: "Введение в теорию множеств и общую топологию".


КУРСЫ ДЛЯ ПРОДВИНУТЫХ МАТЕМАТИКОВ

Математический анализ
У. О. Рудин: "Основы математического анализа".
А. И. Маркушевич: "Теория аналитических функций".
S. Ramanan: "Global calculus".
H. Amann, J. Echer: "Analysis".
W. Fidcher, I. Lieb: "A Course in Complex Analysis: From Basic Results to Advanced Topics".

Дифференциальные уравнения
В. И. Арнольд: “Обыкновенные дифференциальные уравнения”. Книга для уверенных в себе математиков. Диффеоморфизмы, фазовые потоки, гладкие многообразия. Слава Гермесу Трисмегисту!

Теория категорий
С. Маклейн: "Категории для работающего математика".
Р. Голдблатт: "Топосы. Категорный анализ логики".

Геометрия
Д. Мамфорд: "Красная книга о многообразиях и схемах".
К. Номидзу: "Основы дифференциальной геометрии".
J. Lee: "Manifolds and DIfferential Geometry".
L. Nicolaescu: "Lectures on the Geometry".
P. Michor "Topics in Differential Geometry".

Топология
J. Strom: "Modern classical homotopy theory".
T. Dieck: "Algebraic topology".


МАТЕМАТИКА ДЛЯ НЕМАТЕМАТИКОВ:

С. Гроссман, Дж. Тернер: “Математика для биологов”.
Я. Б. Зельдович: “Высшая математика для начинающих и ее приложения к физике”, “Высшая математика для начинающих физиков и техников”.
Г. С. Ландсберг: “Элементарный учебник физики” в трех томах.
М. А. Шубин: “Математический анализ для решения физических задач”.
Я. Б. Зельдович, А. Д. Мышкис: “Элементы прикладной математики“.


ИНТЕРЕСНОЕ:

Цикл “Manga guide to...“. Популярное изложение различных областей математики (и не только), оформленное в виде манги. Увы, без фансервиса.
П. С. Александров: “Введение в теорию групп“. Просто о сложном. Несколько вольный язык изложения, местами затрудняющий восприятие. Но, в целом, must read для начинающих.
В. Б. Алексеев: “Теорема Абеля в задачах и решениях”.
Р. Курант, Г. Роббинс: “Что такое математика?”. Очень интересная книга, в двух словах не описать. Но вас захватит, надолго.
Н. Я. Виленкин: "Рассказы о множествах". Теория множеств для широкого круга читателей.
М. М. Постников: “Теорема Ферма. Введение в теорию алгебраических чисел”.
Н. Стинрод: “Первые понятия топологии“.
А. Я. Хинчин: “Три жемчужины теории чисел“.
О. Я. Виро, О. А. Иванов, Н. Ю. Нецветаев, В. М. Харламов: “Элементарная топология”.
Я. П. Понарин: “Алгебра комплексных чисел в геометрических задачах”.
В. В. Острик, М. А. Цфасман: “Алгебраическая геометрия и теория чисел: рациональные и эллиптические кривые”
В. И. Арнольд: “Вещественная алгебраическая геометрия”
А. А. Заславский: “Геометрические преобразования”.
В. Акопян, А. А. Заславский: “Геометрические свойства кривых второго порядка”.
В. И. Арнольд: “Геометрия комплексных чисел, кватернионов и спинов”.
В. В. Прасолов: “Геометрия Лобачевского”.
В. Г. Сурдин: “Динамика звездных систем”.
Д. В. Аносов: “Дифференциальные уравнения: то решаем, то рисуем”.
В. В. Прасолов: “Наглядная топология”.
Д. В. Аносов: “От Ньютона к Кеплеру”.
М. Клайн: “Математика. Поиск истины“.
Д. Пойа: “Математическое открытие“.
Л. Кэрролл: “Логическая игра“.
Д. Пойа: “Как решать задачу“.
А. Хэтчер: "Алгебраическая топология".
О. Я. Виро, Д. Б. Фукс: "Введение в теорию гомотопий. Гомологии и когомологии".
T. Sundstrom: "Mathematical reasoning writing and proof". Мне кажется отличная книга для первого чтения по математике. В ней объясняется, собственно, что такое математическео доказательство, математический факт и каким образом их можно придумывать. Начала теории множеств.
D. Dummit R. Foote: “Abstract Algebra“. Много примеров, задач, но страшно скучный учебник, его нужно держать как справочник.


ПОЛЕЗНЫЕ РЕСУРСЫ:

Библиотечка "Квант": math.ru/lib/ser/bmkvant
Высшая математика просто и доступно, по 2 курс включительно: mathprofi.net
Необъятная онлайн библиотека: gen.lib.rus.ec

Обсуждаем и дополняем!
Аноним 14/05/16 Суб 23:36:58  362620
Господа, еще раз призываю помочь составить краткое описание для каждой книги в списке. Любые замечания указывайте только с предполагаемым изменением, т е как должно быть. Цель - составить максимально разнообразный список литературы, в котором новичок однако же не потеряется.
Аноним 15/05/16 Вск 00:03:39  362623
>>362620
Маклейн - канонiчноеъ введение в котегории, но русский перевод сука ниоч. Масса опечаток. То домены/кодомены перепутают, то функтор потеряют, то ещё какая беда. В общем, каноном оно остаётся, но для начинающего без некоторого бэкграунда - это адъ, погибель и Израиль. Читать лучше в оригинале всё же. Или попроще - Awodey, "Category theory" (ну как попроще, заскучать и там не выйдет, но у Маклейна совершенно ебанутый стиль). Есть неплохая (ну как неплохая - мне понравилась вроде) Abstract and concrete categories: the joy of cats. Но она немного своеобразно построена. Сначала всё просто, просто, а потом хуяк и расслоения конкретных категорий, алгебры, соответствия Галуа с их декомпозицией, и только потом то, что обычно помещают в начале - объекты и морфизмы. Некоторые упражнения такие же ебанутые, как у Маклейна. К сожалению, талмуд несколько устарел. Например, в части квазикатегорий - нынче продвинутая общественность называет квазикатегориями не совсем то, что подразумевали авторы кошек. С другой стороны в примерах упоминается уйма всяких разных категорий, так что можно составить некоторое представление, о чём идёт речь и как описываемое явление выглядит в конкретной ситуации. Для любителей тапалогий - много примеров из Top, для извращенцев от ИТ - какие-то экзотические Σ-Seq.
Аноним 15/05/16 Вск 00:06:15  362624
Друзья, какой уровень английского требуется для чтения иностранной мат. литературы? Есть книжка, чтобы поднатаскать это?
Срочно нужно.
Аноним 15/05/16 Вск 00:10:22  362626
>>362624
Если это не совсем популярная мат.литература, то важнее понимать птичий язык, а не английский. Самого английского там оказывается не так много и он играет роль прокладки между блоками определений, теорем, доказательств и следствий. Язык обычно достаточно стандартен, благо научный стиль к цветастости и экспрессии не предрасполагает. Вот прямо чтоб срочно - так то хуй знает. С практикой обычно само приходит, а сколько и у кого такая практика отнимет времени - это сугубо индивидуально.
Аноним 15/05/16 Вск 00:18:06  362627
Ну и да, наверно важно преодолеть некий психологический барьер и не бояться каких-то "нереальных" трудностей. Я вот тоже сначала морозился, типа ну как же так, там и так темы непростые, а ещё нипарусски, нипанять ничего, а потом нужда заставила-таки, втянулся и начал хуярить так, что только щепки летят.
Аноним 15/05/16 Вск 00:21:17  362628
>>362618 (OP)
Сука, список говно опять Пиздос
Ramanan -анализ?
ну давай прочитай этот курс на мехмате в первом семестре фреш поступившим
Куча дерьма в списке
Комментарии убогие
Аноним 15/05/16 Вск 00:24:29  362629
>>362628
А я бы сказал, что в тебе куча дерьма.
Аноним 15/05/16 Вск 00:30:43  362632
Сука Вакиль опять в базовых пидоры
Разделить дифгем и алгебраическую геометрию
Дифгем - Раманан, Микор, Ли, Николаеску
Алгем - Вакиль, Мамфорд
К алгему добавить
Bosch Commutative algebra and algebraic geometry
Gotz Wedhorn Algebraic geometry Schemes
Harris Geometry of schemes
Аноним 15/05/16 Вск 00:31:47  362633
>>362629
пруф или балабол
Ты меня нихуя не знаешь, значит балабол
Ты слит, пошел нахуй
Аноним 15/05/16 Вск 00:32:31  362634
>>362632
Другое дело. Еще если описание добавь.
Аноним 15/05/16 Вск 00:34:54  362635
>>362634
Бля, чет язык заплетается. Ну думаю и так понятно что я имел в виду.
Аноним 15/05/16 Вск 00:37:32  362636
14632618520210.gif (1526Кб, 470x353)
КНИГИ ПО МАТЕМАТИКЕ ОПТОМ
Список http://pastebin.ru/0GwfJtXk
Торент http://rgho.st/7rCxBWGhR
Аноним 15/05/16 Вск 00:40:59  362637
Если b^2 делится на n, это же не всегда значит, что и b делится на n? В каких случаях это справедливо, аноны?
Аноним 15/05/16 Вск 00:41:41  362638
>>362637
>это же не всегда значит, что и b делится на n
Всегда.
Аноним 15/05/16 Вск 00:42:07  362639
>>362638
Ну кроме того случая, когда n=b^2.
Аноним 15/05/16 Вск 00:44:10  362640
>>362639
Во, я как раз из-за него и задумался. Думал, может еще какие случаи бывают, а я их не вижу.
Аноним 15/05/16 Вск 01:03:43  362641
есть разделы математики, где функции задают неоднозначные отношения?
Аноним 15/05/16 Вск 01:32:29  362644
Где божественный Атья-Макдональд? Это вообще мастрид, если ты не картофан, задач дохуя интересных, все почти с нуля, для начинающих самое то. В алгем можно добавить Манина Теория схем и квантовые группы.
Аноним 15/05/16 Вск 01:33:08  362645
По общей/теоретико-множественной топологии:
1)V.Runder "A Taste of Topology"(уже было, собственно) - неплохая книга по метрическим пространствам и общей топологии, затрагивает фундаментальную группу. Лишнего мало
2)J.Munkres "Topology" - надо знать, что Munkres по специализации есть теоретико-множественный тополог, то есть книга именно жесткий такой учебник по теоретико-множественной топологии. В том плане, что много ненужного для других областей математики. А так книга ничего
3)Учебник М.Вербицкого по основаниям математики:
http://verbit.ru/MATH/UCHEBNIK/top-book.pdf - специфическая книга, много внимания уделено другим вещам
4)Виро, Харламов, Нецветаев, Иванов "Элементарная топология" - задачник, всё в задачах
5)Crossley "Essential Topology" - учебник-пререквизит для изучения алгебраической топологии. Не затрагивает тему метрических пространств
6)Лекции Хэтчера
https://www.math.cornell.edu/~hatcher/Top/TopNotes.pdf
Краткий ликбез общей топологии перед его уже книгой по алгебраической

Аноним 15/05/16 Вск 01:33:48  362646
>>362645
fix: Runde, а не Runder
Аноним 15/05/16 Вск 01:48:52  362647
Действительный/вещественный анализ
1)Зорич "Математический анализ" - объемный 2-х томник. Первый том посвящен классическому анализу. Много примеров, много материала, в том числе даются в начале основы матлогики и теории множеств, а также функций между ними
2)Pugh "Real Mathematical analysis" - более простая версия Рудина с картинками. Норм книга, но не самая лёгкая
3)Tao "Analysis" - основательный курс анализа для начинающих. Очень фундаментальный
4)Rudin "Principles of mathematical analysis" - хз, нужен ли, если есть Pugh, но может кому больше понравится.
5)Львовский "Лекции по математическому анализу" - записки лекций из НМУ. ОЧЕНЬ ОЧЕНЬ кратко, нужно знать основы калькулюса(интеграл как площадь под графиком, ряды, производные и т.д.)

Аноним 15/05/16 Вск 02:01:27  362648
Алгебра
1)Винберг "Курс алгебры" - нормальный курс на русском для начинающих. Линейная алгебра есть
2)Rotman "Advanced Modern Algebra"(3-е издание, часть первая) - неплохой фундаментальный курс алгебры. Надо знать про индукцию и множества, а также в первых параграфах про элементарную теорию чисел консультироваться с другими учебниками(которые он цитирует) или с интернетом. Линейная алгебра есть
3)Grillet "Abstract Algebra" - очень лаконичный и понятный учебник, но требует немного mathematical maturity(математической культуры). Надо знать элементарную теорию чисел(ту, что в первом-втором параграфах Ротмана, или ту, что в школе учили, я лично в школе нихуя не учил, особенно теорию чисел), про индукцию, про множества и функции(инъекция-сюръекция-биекция-элементарные свойства). Линейной алгебры нету
4)Aluffi "Algebra: Chapter 0" - идея хорошая, а исполнение так себе. Охуенно показывает связь между теорией категорий и собственно алгеброй, сама алгебра изложена так себе, ИМХО. Можно читать параллельно с другим учебником алгебры. Линейная алгебра есть
5)Городенцев "Алгебра. Учебник для студентов-математиков" - кратко, я бы сказал, вырос из лекций НМУ, что как бы намекает. Можно попробовать параллельно с Винбергом(Винберга читать в первую очередь)
6)Roman "Advanced Linear Algebra" - хороший учебник по линалу(алгебраическому, а не по маханию матрицами и определителями над полем действительных чисел). Но нужно знать элементарные свойства матриц и определителей. По ним можно прочитать первую главу книги Artin "Algebra" или начало любой элементарной книги по линейной алгебры)
В середине книги понадобится знакомство с группами, кольцами и их элементарными свойствами
7)Artin "Algebra" - добавляю чисто из-за первой главы, где хорошо рассказывается про матрицы и определители(не в стиле "для дебилов"), и из-за того, что некоторые сравнивают с Винбергом. На сам Артин вообще крутой очень(математик, а не учебник)
Аноним 15/05/16 Вск 02:08:39  362649
Есть уравнение 2a^2=b^2 (диофантово вроде?). Есть задача доказать (через жопу) неразрешимость в целых ненулевых числах a и b.

Набросок того, что мне кажется похожим на доказательство. Из уравнения следует, что 1) 2 | b^2 и 2) a^2 | b^2.

Из 1 следует, что либо i) 2 | b, либо ii) b^2=2. В случае ii b не целое число и доказательство завершается. Поэтому рассмотрим вариант i, который (как и все уравнение целиком) указывает на четность b. Поэтому представим b=2q. Подставим это в уравнение, 2a^2=4q^2, т е a^2=2q^2.

Из 2 следует, что либо iii) a^2 | b, либо iv) a^2=b^2. Случай iv опять не подходит, т к уравнение при этом верно только при a=b=0. Берем случай iii, из которого следует, что 2q^2 | 2q, что не удовлетворяет ни одному целочисленному значению q. А из a^2=2q^2 следует, что при нецелочисленных q, a тоже не может быть целочисленным.

Оцените высер ньюфага, пжлст. Только не сильно обижайте.
Аноним 15/05/16 Вск 02:11:21  362650
Классическая линейная алгебра(которая одновременно излагается и в некоторых учебниках по общей алгебры), без модулей и некоторых чисто алгебраических аспектов:
1)Гендальф, то есть Гельфанд "Лекции по линейной алгебре" - хз, что сказать. Книга не без минусов, ни и не без плюсов. Круче всяких Ильинов Поздняков и прочих Беклемишевых. НО! Не даётся определение определителя
2)Кострикин-Манин "Линейная алгебра и геометрия" (довольно кратко, затрагивается темы геометрий и связей с квантовой механикой). Не даётся определение определителя
1)Axler "Linear Algebra Done Right" - подход без определителей(почти). Одна из самых популярных книг за рубежом.
2)Treil "Linear Algebra Done Wrong" - не такая популярная, как Axler, но тоже хвалят, да. Определители есть.
3)Shilov "Linear Algebra" - хз, что сказать, некоторыехвалят, определитель появляется на первой странице
4)Hoffman-Kunze "Linear Algebra" - классика за рубежом. Вроде норм
5)Halmos "Finite-Dimensional Vector Spaces" - хвалят те, кто хвалит Hoffman-Kunze, тоже классика
6)Peterson "Linear Algebra" - не особо знаком, но выглядит аккуратно. Что-то вроде Акслера
7)Roman "Advanced Linear Algebra" - см. секцию про алгебру. Можно читать второй книгой после элементарной книги по линалу из списка выше.
Аноним 15/05/16 Вск 02:19:46  362651
>>362649
Я бы так решил.
a,b - целые, не равные нулю.
2a^2=b^2 => 2=(b/a)^2=p^2, где p - рациональное число.
Но, так как нет рационального числа, квадрат которого равен 2 следует, что нет и такой пары целых чисел b/a=p.
Аноним 15/05/16 Вск 02:22:25  362652
>>362651
Да, так проще. Просто задача как раз и состоит в том, чтобы доказать что корень из двух не является рациональным числом. Вот я и подумал, что ссылаться в доказательстве на сам вопрос как-то неловко.
Аноним 15/05/16 Вск 02:25:12  362654
>>362652
А я думал, тебе уравнение решить надо.
Аноним 15/05/16 Вск 02:26:42  362655
>>362654
А с доказательством-то я не обосрался часом?
Аноним 15/05/16 Вск 02:51:26  362659
14632698862500.png (4Кб, 390x190)
>>362655
Обосрался
>Из уравнения следует, что 1) 2 | b^2 и 2) a^2 | b^2.
Следует, что 2a^2| b^2, но не следует, что каждый из{2;a^2} делит b. (a | b - означает, что b - делитель a, так? Сам просто почти не использую это обозначение).

Вот тебе моё решение тогда. Предполагаем, что a и b не имеют общих делителей. f(a;b)=НОД(a;b). А потом из предположения, что a,b - целые и свойств делимости, доказываем, что НОД(a;b)=2(можно было продолжить и доказать, что он равен 2^n), что противоречит начальному предположению. .
Аноним 15/05/16 Вск 02:52:49  362660
14632699695520.png (4Кб, 390x161)
>>362659
Лол. Зачем я дробь нарисовал? Вот исправленный вариант.
Аноним 15/05/16 Вск 03:03:33  362665
>>362659
>(a | b - означает, что b - делитель a, так?
Наоборот, a делит b.

Аноним 15/05/16 Вск 03:09:31  362666
>>362665
Блин, с отправкой поста просрался.

>>362659
>Следует, что 2a^2| b^2, но не следует, что каждый из{2;a^2} делит b.
Почему? Разве не ты в треде мне говорил, что если число делит квадрат другого числа, то оно всегда делит и его самого, кроме случая, когда оно равно этому квадрату?

>Вот тебе моё решение тогда.
Угу. По-моему, это доказательство, которое еще Эвклид приводил. Задача найти другое решение, поэтому я и указал в скобочках, что доказать надо через жопу.
Аноним 15/05/16 Вск 07:02:34  362682
Анон, как доказать первый замечательный предел без формулы площади сектора?
Аноним 15/05/16 Вск 07:42:04  362684
>>362650
> 1)Axler "Linear Algebra Done Right"
Вот эта нормас, я ее прочитал и прорешал.
Аноним 15/05/16 Вск 08:03:21  362687
14632886015580.png (146Кб, 889x772)
Поясните за пикрелейтед. Почему во втором случае $d$ делится на $\psi$? Я так понял, это следует из того, что если $p$ --- многочлен, $p^{(n)} \neq 0$, но $p^{(n)}(c) = 0$, то $c$ --- корень $p$ кратности не меньше $n$. Доказал я это так. Сначала по индукции доказал, что
$$p'(z) = \left[\prod(z - c_i)\right]' = \sum \pi_i,$$
где
$$\pi_i = \frac{\prod(z - c_i)}{(z - c_i)}.$$
Отсюда видно, что при каждом дифференцировании пропадает одна скобка, т. е. кратность корня уменьшается на 1.
Получается, что кратность $\lambda_i$ как корня $d$ не меньше $n_i - 1$, так как $d^{(n_i - 1)}(\lambda_i) = 0$.

Так вот, у меня 2 вопроса: верно ли мое рассуждение про многочлены и верно ли, что подчеркнутое утверждение в теореме действительно следует из него? Или может как-то все намного проще? Может автор считает эту хуйню про многочлены очевидной, но мне несколько часов понадобилось, чтобы додуматься.

Спасибо.
Аноним 15/05/16 Вск 09:05:05  362695
>>362687
Поясню еще. Когда $n$ раз дифференцируем многочлен, получается сумма всех возможных произведений $\deg p - n$ скобок. По индукции кратность $c$ не меньше $n$, поэтому все произведения кроме, быть может, того, из которого $n$ раз убрана скобка $(z - c)$, равны 0. Тогда то произведение тоже не равно 0, а значит, кратность $c$ не меньше $n + 1$.
Аноним 15/05/16 Вск 09:27:43  362698
>>362687
Мне тоже не очевидно 3:
Аноним 15/05/16 Вск 09:40:56  362699
>>362687
По-моему там хуйня написана
Возьмём А=
1 1 0 0
0 1 1 0
0 0 1 0
0 0 0 2
P=(x-1)(x-2)
Q=(x-1)^3 (x-2)
спектр (А) = {1,2} на спектре P и Q совпадают (и равны нулю) но P(A) != 0 и Q(A) = 0, ЧЯДНТ?
Аноним 15/05/16 Вск 09:54:26  362701
>>362620
Блять, даже я потерялся, а я кандидатскую скоро защищать буду.
Аноним 15/05/16 Вск 09:55:21  362702
>>362701
Кандидатскую по математике? Бедненький...
Аноним 15/05/16 Вск 09:55:48  362703
>>362702
>:
Аноним 15/05/16 Вск 10:16:44  362704
>>362650
Обосрался от характеризация книг исключительно по наличию определителя.
Аноним 15/05/16 Вск 10:38:22  362705
>>362704
Ну а что, эта классификация ничем не хуже других классификаций.
Аноним 15/05/16 Вск 11:08:03  362714
>>362699
"Значения на спектре совпадают" в этой книге означает, что и производные до определенного порядка тоже совпадают.
Аноним 15/05/16 Вск 11:08:50  362715
Анон, мне нужен учебник по математике чтобы вспомнить базовую программу в школе и идти дальше. Школу закончил 2 года назад, 2 месяца проучился в техническом вузе, за эти месяцы учил только матрицы. В общем, школьную программу я частично помню, мне больше нужно подтянуть общую теорию, чтобы двигаться дальше. какой учебник мне взять, чтобы не учить всю школьную программу заново?
Аноним 15/05/16 Вск 11:10:00  362716
>>362699
>>362714
То есть в твоем примере P'(1) и P''(1) должны быть равны 0, потому что размер жордановой клетки равен 3.
Аноним 15/05/16 Вск 11:33:13  362721
14633011940090.jpg (69Кб, 850x400)
Elements of Abstract and Linear Algebra
Edwin H. Connell

На мой взгляд это (почти) идеальная первая книга по алгебре, да и математике вообще.

Алгебра – 1
А.Л. Городенцев

Заебись, к нему могут быть претензии с педагогической точки зрения, но с точки зрения содержания он прекрасен.

Основные понятия алгебры.
И.Р. Шафаревич

Замечательный обзор вообще того, что такое алгебра, как она выглядит и какое место она занимает в математике. Примеры, приложения и прочая конкретика.

Конкретная теория групп I: основные понятия.
Николай Вавилов.

И вообще все остальные книги (и лекции!) Вавилова. Насчёт его стиля и идеологии ещё могут быть претензии, но это охуенная книга. Чего стоят только биографические описания математиков.
Аноним 15/05/16 Вск 11:38:45  362724
>>362721
С Вавиловым история крайне мутная.
http://lj.rossia.org/users/kouzdra/714455.html
Аноним 15/05/16 Вск 11:40:11  362726
>>362682
По Лопиталю
Аноним 15/05/16 Вск 11:51:32  362734
>>362724
«История крайне мутная» означает одно сомнительное предложение в одной из книг? К тому же в более свежей версии (это всё получерновики) формулировка исправлена.
Аноним 15/05/16 Вск 11:53:01  362735
>>362734
А где гарантия, что больше лажи нет?
Аноним 15/05/16 Вск 12:18:17  362738
>>362735
1. К книгам никто гарантии не выдаёт. Тем более, в математике. ТЫ САМ ВИНОВАТ, ЧТО ПОВЕРИЛ
2. В книгах по алгебре нет, это точно.
Аноним 15/05/16 Вск 12:37:55  362744
>>362695
>>362687
Короче, посаны, вот че я понял.
Это все дело следует из общего утверждения о многочленах.
Пусть $p$ --- многочлен над $\mathbb{C}$, $\deg p > n$, $c$ --- корень $p$. Обозначим кратность $c$ как $\mu(c)$. Тогда
$$p(c) = p'(c) = \dots = p^{(n)}(c) = 0$$
в том и только том случае, если $\mu(c) \geq n + 1$.
Аноним 15/05/16 Вск 12:38:35  362745
>>362744
Так и вот, это что какой-то общеизвестный факт? У него есть название?
Аноним 15/05/16 Вск 12:52:52  362750
>>362618 (OP)
Это Гаусс, Гёдель, Кокс(е)тер?, Перельман??
Аноним 15/05/16 Вск 13:06:52  362752
Всем привет. Задумался я подтянуть матан дабы быть не просто джава-макакой, а попробовать ещё и в машинное обучение. В треде по ML мне посоветовали Зорича. Подойдёт ли?
Аноним 15/05/16 Вск 13:11:33  362754
>>362750
>Гаусс
Да.
>Гёдель
Да.
>Кокс(е)тер?
Жорж де Рам.
> Перельман??
Григорий. Да.
Аноним 15/05/16 Вск 13:13:30  362756
Ну что за даун переатывал? Сказали же, куча книг невнятных в описании, а он тупо все копирует. Ебаные вербитки с выбегалами.
Аноним 15/05/16 Вск 13:19:15  362760
>>362754
>Жорж де Рам
Ооо, а у него гомологические токи и теория поля!
Аноним 15/05/16 Вск 14:14:52  362769
>>362752
> В треде по ML мне посоветовали Зорича.
В принципе норм, но слабоватый учебник.
Читай лучше "Бурбаки: Основные структуры анализа".
Аноним 15/05/16 Вск 14:27:06  362770
>>362756
>Ну что за даун
Судя по всему, папа твой постарался. Его почерк.
Аноним 15/05/16 Вск 16:15:41  362786
Докажите существование расходящейся последовательности.
Аноним 15/05/16 Вск 16:20:53  362791
>>362786
Существование натуральных чисел подвергают сомнению в соседнем треде.
Аноним 15/05/16 Вск 16:25:05  362792
>>362791
Так я ж не подвергаю сомнению. Просто хочу увидеть доказательство.
Аноним 15/05/16 Вск 16:27:33  362793
подскажите задачник по алгебре(универ)
Аноним 15/05/16 Вск 16:28:32  362794
>>362792
Доказательство в какой аксиоматике? Ты принимаешь ZFC?

>>362793
Проскуряков.
Аноним 15/05/16 Вск 16:29:09  362795
>>362794
Да без проблем.
Аноним 15/05/16 Вск 16:41:00  362799
>>362795
Пусть существует по крайней мере одно индуктивное множество. Тогда в нём существует наименьшее по включению индуктивное подмножество. Оно удовлетворяет аксиомам Пеано. Таким образом, натуральные числа существуют.

Рассмотрим последовательность 1, 2, 3, ...
Известно, что последовательность вещественных чисел является сходящейся тогда и только тогда, когда она является последовательностью Коши. Последовательность 1, 2, 3, ... не является последовательностью Коши, так как расстояние между двумя разными точками последовательности по меньшей мере равно 1.

Таким образом, последовательность 1, 2, 3, ... расходится.
Аноним 15/05/16 Вск 16:51:39  362802
>>362799
Не, ты построил пример.
Аноним 15/05/16 Вск 16:55:43  362804
>>362802
Блядь. Ненавижу тебя.
Аноним 15/05/16 Вск 17:05:50  362805
Я просто отставлю это здесь

>Но происходят и психологически фундаментальные изменения. Сейчас эти изменения происходят в форме сложных теорий и теорем, при которых оказывается, что замещением старого образа-структуры, например, натуральных чисел, служит некоторый правополушарный образ. Вместо множества, рассыпанного на элементы, мы наблюдаем какие-то смутные пространства, которые могут очень сильно деформироваться, отображаться друг в друга, причем каждый раз конкретное пространство не важно, а важно только пространство с точностью до деформации. Если мы очень хотим вернуться к дискретному образу, мы рассматриваем непрерывные компоненты, те куски, из которых это все состоит. Раньше все эти пространства возникали как собранные из канторовских множеств, потом были отображения между ними, собранные из канторовских отображений, потом гомотопии и т.д. Это была довольно сложная лестница, и множества были внизу. На мой взгляд – это надо проработать, я в этом уверен довольно сильно, но не на сто процентов, в общественном сознании сейчас происходит переворот: низом становится правополушарная картина мира, гомотопическая, а если вы хотите говорить в дискретных терминах, то вы производите факторизации. То есть канторовские точки стали не точками, а, скорее, аттракторами, областями притяжения, непрерывными компонентами и так далее – с самого начала. Канторовская проблема бесконечности перестает быть актуальной: оно все с самого начала настолько бесконечно, что если вы хотите из него изготовить что-то конечное, то вы его должны очень сильно ужать.

>Кстати, это параллельно тому, как мы обращаемся с фейнмановскими интегралами. Когда берешь физическое определение фейнмановской формулы, то первые два, три, четыре шага все формулы не имеют смысла. Сначала фейнмановский интеграл, он никак не определен. Потом ряд теории возмущений, который не просто расходится, а у него еще каждый член бесконечен. Потом регуляризируется каждый член, и каждый член становится конечным, но ряд, как правило, все равно расходится. Потом вы интерпретируете ряд. И, наконец, пройдя через эту серию бесконечностей, вы получаете финитный ответ. И таким способом была получена серия замечательных математических теорем. Я наблюдаю в этом аналог перестройки математики в терминах теории категорий и гомотопической топологии.

>Юрий Манин
Аноним 15/05/16 Вск 18:06:06  362821
>>362704
C линейной алгеброй действительно нужны пояснения. Очень много учебников с совершенно разным набором тем.
Аноним 15/05/16 Вск 18:06:06  362822
>>362735
А где там лажа? Вижу интуитивное описание аксиоматического подхода на пальцах, можно говорить только о том, насколько оно кривое/прямое. Тащемта, полностью согласен с kkpp в комментах оттуда же.
Аноним 15/05/16 Вск 18:23:39  362829
>>362827
https://ru.wikipedia.org/wiki/%D0%9C%D0%B0%D0%BD%D0%B8%D0%BD,_%D0%AE%D1%80%D0%B8%D0%B9_%D0%98%D0%B2%D0%B0%D0%BD%D0%BE%D0%B2%D0%B8%D1%87
Аноним 15/05/16 Вск 19:06:16  362856
>>362821
Бери Treil. Он простой, почти "дубовый", весь необходимый набор джентльмена у него есть. Потом уже наворачивай курсы по общей алгебре, где будешь ворочать модулями.
Аноним 15/05/16 Вск 19:24:07  362867
Аноны, а что вы думаете про WTF (великая теорема ферма)?
Аноним 15/05/16 Вск 19:30:17  362869
>>362867
Fermat's Last Theorem. Она доказана.
Аноним 15/05/16 Вск 19:53:43  362875
>>362867
Великую теорему Ферма доказал в 1994 году Эндрю Уайлс.
В 2016 году он получил за это доказательство премию Абеля(можно сказать, самую престижную премию по математике). Он и другие премии получал, но эта - самая выдающаяся.

Но есть выбегаллы с dxdy и провинциальных мехматов, которые либо не знают о том, что ВТФ доказана, либо считают, что можно найти простое доказательство методами элементарной теории чисел. Естественно, никто из этих "одарённых" ни к чему не продвинулся.

Но на dxdy(которые якобы борятся с антинаукой) пошли дальше и дальше этим целый отдельный раздел.
Аноним 15/05/16 Вск 19:54:51  362876
>>362875
>дальше этим целый отдельный раздел.
fix: не дальше, а дали
Аноним 15/05/16 Вск 20:55:57  362931
Наконец-то годный математический тред. Я все понимаю о чем речь.
Аноним 15/05/16 Вск 22:57:03  363001
>>362769
лучше уж Шварц тогда
Аноним 15/05/16 Вск 23:04:17  363006
Помогите полному нубу. Есть значение некоторой функции в нескольких точках. Надо восстановить саму эту функцию. Предполагаю что способов очень много, но нужен тот, который кладет прибор на точность, но при этом как можно менее затратен с точки зрения вычислений(будет считать древний микроконтроллер) Какой способ мне гуглить?
Аноним 15/05/16 Вск 23:09:59  363009
Аноны, кто как читает книги? Параллельно конспект ведете, или выжимки какие-то?
Аноним 15/05/16 Вск 23:19:25  363017
>>363009
Записываю в тетрадь определения и доказательства.
Аноним 15/05/16 Вск 23:21:01  363018
>>363017
Доказательства по ходу переписываешь, или читаешь - воспроизводишь?
Аноним 15/05/16 Вск 23:25:04  363019
>>363018
Сначала пытаюсь сам доказать. Если не могу читаю доказательство из учебника.
А в конспект переписываю, чтобы лучше запомнить.
Самими конспектами почти не пользуюсь проще в учебнике посмотреть, они нужны чтобы в голове отложилось, лол.
Аноним 15/05/16 Вск 23:56:04  363029
>>363019
Ты крут, попробую так же.
>они нужны чтобы в голове отложилось, лол
Да! Вот как бы это у себя в голове провернуть так.
Сколько времени уделяешь в среднем? Все индивидуально, но все же.
Аноним 16/05/16 Пнд 00:10:02  363033
>>363009
Веду небольшие конспекты вида "уместить 400 учебник в 2 листа А4" в которых основные записываю основные теоремы. Иногда перечитываю и пытаюсь восстановить детали доказательств, если тяжко - то подсматриваю. Стараюсь прорешать каждое упражнение и каждую теорему воспринимаю как упражнение, если тяжко - то подсматриваю или забиваю хуй с надеждой вернуться потом. Пытаюсь самостоятельно придумывать примеры к определениям из тех, которых нету в учебнике. Вот.
Аноним 16/05/16 Пнд 00:20:45  363035
>>363033
Можешь выложить что-нибудь, если не сложно? 400 страниц в 2 листа - это надо бы увидет.
>Пытаюсь самостоятельно придумывать примеры к определениям из тех, которых нету в учебнике.
Error: can't compile. Please repeat.
Аноним 16/05/16 Пнд 02:33:48  363063
>>363035
Да они у меня хаотичны, всегда не дописаны, хуёво свёрстаны и вряд ли кому-то представляют какой-то интерес. Ну вот по гильбертовым пространствам например: http://rgho.st/8S6CDWCbB
Аноним 16/05/16 Пнд 02:34:09  363064
>>363063
Кстати, раз уж выложил, то рейт с конструктивной критикой - кто в теме.
Аноним 16/05/16 Пнд 03:22:01  363068
>>363063
"Записки сумасшедшего математика"
Про тему не в теме, но идея выписывать только основные теоремы/замечания вроде годная. В "элементарной топологии" Виро такой же формат, и формулировки лучше запоминались почему-то. Обычно в один глаз влетает и не долетает.
Аноним 16/05/16 Пнд 03:42:10  363073
Как быть таким же охуенным, как Гаусс или Перельман?
Аноним 16/05/16 Пнд 03:43:05  363074
>>363073
Въёбывать.
Аноним 16/05/16 Пнд 03:45:26  363075
Посоветуйте книги по аналитической геометрии.
Аноним 16/05/16 Пнд 03:50:20  363076
>>363075
ебаш сразу линал
Аноним 16/05/16 Пнд 04:18:16  363079
>>363068
Ты про Виро Харламов Иванов? Уж не сказал бы что теоремы и замечания там только основные - мне кажется у него дико перегруженныый учебник, который можно читать только как паблик "прикольных фактов из общей топологии", типа "А вы знали, что есть Т3 пространство у которого нет непрерывных функций в R - отличных от констант?", не знал, прикольно, но пережил бы и если бы никогда не узнал. Читать его линейно проделывая все упражнения - не самая эфективная трата времени, имхо.
Аноним 16/05/16 Пнд 06:13:58  363085
>>362618 (OP)
Было бы неплохо, запаковать всё это в торрент, с ОП-постом в html-ке. А сюда - магнет-линк.
Аноним 16/05/16 Пнд 06:16:09  363087
>>363085
Там сириусли плохой список, ОПу уже много раз об этом говорили (не я, я сейчас говорю). Лучше возьми список какого-то одного задрота и ебаш по нему.
Аноним 16/05/16 Пнд 06:47:27  363094
>>363087
>возьми список какого-то одного задрота
Например?
Аноним 16/05/16 Пнд 06:48:20  363095
>>363094
Вербицкий.
Аноним 16/05/16 Пнд 06:54:36  363099
14633708766920.png (1Кб, 328x65)
Котаны, помогите долбаёбу, позабывшему ТФКП выделить действительную и мнимую часть полинома
Аноним 16/05/16 Пнд 06:56:04  363100
>>363099
На dxdy, студент! Мы тут тапалогии и Рыбникова обсуждаем!
Аноним 16/05/16 Пнд 06:57:56  363102
>>363099
чё эта за значки блять
Аноним 16/05/16 Пнд 07:03:18  363104
14633713988580.jpg (187Кб, 960x720)
>>363100
я инженер, рили, решил вспомнить ТАУ. Проверяю устойчивость системы по частотному критерию Михайлова. Для построения годографа нужно выделить действительную и мнимую части выражения
Аноним 16/05/16 Пнд 07:14:45  363108
>>363104
1-е Действительная -U, мнимая V
2-е Мнимая - хуйня в скобках, действительная kk_r + T_r
Аноним 16/05/16 Пнд 07:15:34  363109
>>363108
>-U
Действительная - U
Это не минус, а тире(или как там эта хуйня называется).
Аноним 16/05/16 Пнд 07:20:53  363112
>>363108
ты мне объясняешь что есть что в выражениях, которые я написал? Найс, но я и сам это прекрасно понимаю, мне нужно второе выражение привести к виду первого, т.е. найти U(w) и V(w). Я так понимаю, нужно умножать на комплексно-сопряженное число?
Аноним 16/05/16 Пнд 07:24:10  363113
>>363112
V - хуйня в скобках, умножанная на омегу
U=const=kk_r+T_r
Аноним 16/05/16 Пнд 07:28:51  363114
14633729312930.png (22Кб, 849x345)
>>363113
так не бывает, обе части должны быть функциями от w. Значит, нужно на что то умножать, где эта самая w присутствует. Вот пример из пособия, но это преобразование применяется к передаточной функции (являющейся дробью).
Аноним 16/05/16 Пнд 07:32:57  363115
>>363114
либо я облажался с выводом характеристического полинома, сейчас еще раз пробегусь по решению
Аноним 16/05/16 Пнд 07:33:03  363116
>>363114
Ну домнож на сопряженную и раздели на неё.
Аноним 16/05/16 Пнд 08:07:41  363121
14633752614310.jpg (478Кб, 2301x1535)
Прошу помощи с следующей задачей: нужно доказать что множества X и Y равномощны, построим взаимно однозначное соответствие.

X: (-6;11)
Y: [-3;2]

Я знаю как это сделать для [-6,11], но для (-6,11) нет.
Аноним 16/05/16 Пнд 08:10:38  363122
>>363121
Попробуй установить биекцию каждого множества с R.
Аноним 16/05/16 Пнд 08:15:32  363123
>>363122
И потом по свойству что если одно биекиивно R, и второе биективно R, они биективны между собой?
Я как понял из условия нужно установить биекцию именно между ними.
Аноним 16/05/16 Пнд 08:27:03  363125
>>363123
Да.
Аноним 16/05/16 Пнд 08:51:54  363126
>>363095
Там нет книг, там просто темы.
http://imperium.lenin.ru/~verbit/MATH/programma.html
Аноним 16/05/16 Пнд 08:53:40  363127
>>363126
>Список полезных книжек по математике
• Первый курс ◦ Анализ" Лорана Шварца, "Анализ" Зорича,
◦ "Задачи и теоремы из функ. анализа" Кириллова-Гвишиани
◦ Дифференциальная топология (Милнор-Уоллес),
◦ Комплексный анализ (Анри Картан), Комплексный анализ (Шабат)

• Второй курс ◦ Группы и алгебры Ли (Серр)
◦ Алгебраическая топология (Фукс-Фоменко),
◦ "Векторные расслоения и их применения" (Мищенко)
◦ "Характеристические Классы" (Милнор и Сташеф)
◦ "Теория Морса" (Милнор),
◦ "Эйнштейновы Многообразия" (Артур Бессе),
◦ Коммутативная алгебра (Атья-Макдональд),
◦ Введение в алгебраическую геометрию (Мамфорд)
◦ Алгебраическая геометрия (Гриффитс и Харрис),
◦ Алгебраическая геометрия (Хартсхорн)
◦ Алгебраическая геометрия (Шафаревич)
◦ Алгебраическая теория чисел (ред. Касселс и Фрелих)
◦ Теория чисел (Боревич-Шафаревич)
◦ Когомологии Галуа (Серр)
◦ "Инварианты классических групп" (Герман Вейль)

• Третий курс ◦Бесконечнократные пространства петель (Адамс)
◦К-теория (Атья)
◦Алгебраическая топология (Свитцер)
◦Анализ (Р. Уэллс)
◦Формула индекса (Атья-Ботт-Патоди, сборник Математика)
◦Гомологическая Алгебра (Гельфанд-Манин)
◦Когомологии групп (Браун, что ли)
◦Когомологии бесконечномерных алгебр Ли (Гельфанд-Фукс)
◦Кэлеровы многообразия (Андрэ Вейль)
◦Квазиконформные отображения (Альфорс)

• Четвертый курс ◦Геометрическая топология (Сулливан)
◦Этальные когомологии (Милн)
◦Алгебраическая геометрия - обзор Данилова (Алгебраическая Геометрия 2, ВИНИТИ)
◦Группы Шевалле (Стейнберг)
◦Алгебраическая К-теория (Милнор)
◦Обзор Суслина по алгебраической К-теории из 25-го тома ВИНИТИ
◦Многомерный комплексный анализ (Гото-Гроссханс)
◦То же по книжке Демайи (перевод готовится)

• Пятый курс ◦Громов "Гиперболические группы"
◦ Громов "Знак и геометрический смысл кривизны"

Аноним 16/05/16 Пнд 09:12:18  363128
>>363126
Не тот же, это программа когда он с гарварда на понтах приехал http://verbit.ru/Job/HSE/Curriculum/all.txt вот реальная
Аноним 16/05/16 Пнд 09:15:59  363130
>>363127
>>363128

По одной книжке, что-ли искать и качать?
Аноним 16/05/16 Пнд 09:17:41  363131
>>363130
Если ты книжку читаешь/просматриваешь дольше 5 минут, то не вижу проблемы потратить в промежутках 40 секунд на вбивание названия в gen.lib.rus.ec
Аноним 16/05/16 Пнд 09:27:38  363133
>>363131
Вот, терь ясно.
Аноним 16/05/16 Пнд 09:39:56  363136
>>363125
Мне нужно биекцию между X и Y установить напрямую
Аноним 16/05/16 Пнд 09:41:05  363137
>>363136
X<->R
Y<->R
X<->R<->Y
profit?
Аноним 16/05/16 Пнд 09:51:36  363139
>>363121
Это баян
http://math.stackexchange.com/questions/213391/how-to-construct-a-bijection-from-0-1-to-0-1?lq=1
Потом берешь биекцию с (-6; 11) на (0; 1) и с [-3; 2] на [0; 1]; берешь композицию биекций.
Аноним 16/05/16 Пнд 10:09:58  363146
>>363137
Да через биекцию с R это сделать элементарно, нужно напрямую соответствие между Y и X установить, очевидно же
Аноним 16/05/16 Пнд 10:11:21  363147
>>363146
И то и другое просто и используют, по сути, один и тот же трюк.
Аноним 16/05/16 Пнд 10:13:06  363148
14633827866150.png (420Кб, 517x701)
>>363139
А через построение функции нельзя получить?
На ОХ берем (-6;11)
На ОУ [-3;2]
Отмечаем точки пересечения и строим прямую.
Считаем значение линейной функции.
Или я где-то проебался?
Аноним 16/05/16 Пнд 10:17:04  363156
>>363148
Между ними не может существовать биективного непрерывного отображения.
Аноним 16/05/16 Пнд 10:33:37  363163
>>363156
Чому? Точек там вроде одинаково, не?
Аноним 16/05/16 Пнд 10:35:50  363164
>>363163
Потому что интервал не компакт, а по теореме Александрова: биективное непрерывное отображение компакта - гомеоморфизм.
Аноним 16/05/16 Пнд 10:53:03  363168
>>363148
Я вообще не понял о чем ты говоришь. Если можешь, то получи. Я ж тебе сказал нормальный очевидный способ.
Аноним 16/05/16 Пнд 13:54:24  363191
>>363079
Да, про него. Согласен, но чисто зрительно легче воспринимается. Когда сдаешь экзамен и вспоминаешь, что вот это было написано внизу страницы, а справа такой-то рисунок, чтот типо того.
Аноним 16/05/16 Пнд 14:13:47  363192
>>363079
Куда там, уж лучше интегральчики несобственные посчитать, как все мехматовские гении.
Аноним 16/05/16 Пнд 14:53:11  363197
>>363192
Лил, ты правда думаешь, что то, что написанно в виро харламове невъебенно высокая математика недоступная мехматовским картофаном? У тебя Вербитоз.
Аноним 16/05/16 Пнд 15:29:03  363208
>>363085
Йоу, смотри сюда брателла >>362636
Большинство из этого и куда болие лучие книги есть в маём уютном торрентике.
Аноним 16/05/16 Пнд 15:33:33  363210
>>363208
Омг, ты принес это из /pa/?
Аноним 16/05/16 Пнд 15:48:46  363213
>>363210
Это же знаменитый Мальта. Его все знают. Конкретно на твоём пике "Озорница".

мимо олдфаг
Аноним 16/05/16 Пнд 16:09:40  363221
>>363213
Зачем-то удалили
Аноним 16/05/16 Пнд 16:19:39  363228
>>363197
Скажу даже проще - мехматяне туда в принципе не зайдут, так как математика у них будет вызывать отвращение и жжение.
Аноним 16/05/16 Пнд 19:04:21  363257
>>363197
Ну они ее проходят, только почему-то неосознанно. На мехмате принято, что математика наука интуитивная и определения это нечто приходящее после множества частностей.
Аноним 16/05/16 Пнд 19:45:33  363269
>>363257
математика толька для вычислений и физеки нужна, гыыыы) пади возьми интегральчик, маня)
Аноним 16/05/16 Пнд 20:46:45  363273
>>363257
>>363228
Пиздец вы все тут знатоки мехмата я гляжу.
Аноним 16/05/16 Пнд 21:12:33  363277
14634223538360.jpg (58Кб, 850x478)
Стоит по мере прочтения книг сюда постить задачки (не интегральчики конечно, а доказать что-либо) которые не могу решить?
Аноним 16/05/16 Пнд 21:16:56  363278
>>363277
давай
Аноним 16/05/16 Пнд 22:32:27  363281
>>363063
Давайте создадим учебную группу и будем совместно вести конспект математических наук в расшаренном документе.
Аноним 16/05/16 Пнд 22:35:55  363282
>>363281
я за
мимокун
Аноним 16/05/16 Пнд 22:57:45  363286
Давайте захватим мехмат, и устроим там первокультурную диктатуру(выкинем интегральчики и ряды, добавим когомологий, да побольше)
А то охуели они
Аноним 16/05/16 Пнд 23:37:32  363292
Аноны-математики, читаем:
http://dxdy.ru/topic97264-90.html
http://dxdy.ru/topic97264-105.html

Чуваки с мехмата поясняют, что математики нахуй не нужны государству, и поэтому нужно готовить только преподов для технарей или экономистов.
Аноним 17/05/16 Втр 01:43:36  363306
>>363292
>что математики нахуй не нужны государству
А что, это у кого-то вызывает сомнения?
Аноним 17/05/16 Втр 01:44:49  363307
>>363286
нинад
мимомехмат
Аноним 17/05/16 Втр 01:50:10  363308
14634390104530.png (15Кб, 347x335)
Поцыки, поясните за проективную плоскость. Там чет нахуеверчено непонятно что. Вот значит берём лист Мёбиуса и склеиваем его край с краем диска. Тут вроде понятно. Конструкция получается странная и представить сложновато, но сам процесс прозрачен. А вот что там с какими-то несобственными прямыми и отождествлением диаметрально противоположных точек на сфере - вообще пиздос какой-то.
Аноним 17/05/16 Втр 01:50:39  363309
>>363306
Блять, я ещё думал, писать пометку "для дебилов", или не писать. Решил не писать, неужели зря?

Пометка "для дебилов": адекватному человек совершенно поебать, чего там ебаному государству нужно, а если он начинает об этом рассуждать, то он ебаный промытый совок, чмо, и урод, и желает зла и уничтожения всему прекрасному.

Кстати, встречный вопрос для таких совков(риторический, пусть подумают, но не отвечают): а нахуя математике государство(особенно такое)? Почему ты, совок ебаный, пытаешься угодить ебаному государству, а оно на тебя срать хотело.

Прошу прощения за мат. Такие дела, анон.
Аноним 17/05/16 Втр 01:56:51  363312
>>363309
Всё правильно сказал.
Аноним 17/05/16 Втр 02:07:17  363314
>>363309
Я совок ебаный, поэтому я анально одгородился и сижу на dxdy
Аноним 17/05/16 Втр 02:40:58  363317
>>363309
> это школярское подражание тифарету
Найс трай только ты не на своей параше и ни на кого этот "антиавторитарный" еджи понос в 2016 впечатления уже не производит. Самое главное, ты даже не понял что в вопрос "нужна кому-то матёшка или нет" никакого морального осуждения я не вкладывал, просто это очевидный факт. Это государство тебе выделяет деньги, а не ты ему. Это оно может легко прикрыть твою сраную вышечку и покровительствовать рейдерскому захвату мцнмо, а ты ему нихуя сделать не сможешь, ибо выхлопа с тебя ноль и даже если ты сумеешь свалить из сраной рашки никто ничего не потеряет. Ты, как и любой бюджетник, которым ты по сути и являешься, на птичьих правах всегда и везде, поэтому реально единственная правильная для тебя стратегия это быть потише, а своим визгливым пафосом ты добьёшься только того что никто из "быдла" не встанет на твою сторону когда что-то случится.
Аноним 17/05/16 Втр 02:43:27  363318
>>363317
какую парашу ты имеешь ввиду? тифаретник?
Аноним 17/05/16 Втр 02:45:02  363319
>>363317
>это система, тут ничаво зделать низя!!!!
Ясно.
Аноним 17/05/16 Втр 02:54:26  363320
>>363319
И что ты можешь сделать, школьник?
Аноним 17/05/16 Втр 04:34:12  363323
>>363309
Этот твой пост ярко показывает насколько ты далек от науки, а привязывая слово "совок" ты еще ярче показываешь, что в принципе не понимаешь и не знаешь как устроена мировая наука.
Во всем мире наука держится на государстве и самая (одна из) государствозависимая область - это математика. /thread
Государство решает не только сколько давать еды и бабла, но и нужна ли математика вообще. Как напрямую, так и косвенно, через рычаги влияния на вузы, вроде критериев для всяких ранкингов, выделения финансирования для той или иной области, установки KPI и прочего.
Нахуя математике государство? Вопрос элементарный: без государства математики как профессии не будет. И так в любом государстве, будь то "совок" или США. Без него твой путь в макакокодеры + по вечерам формулки дрочить как хобби.
То что математика нахуй не нужна государству - это очевидный факт.
Аноним 17/05/16 Втр 05:00:44  363326
>>363323
>Государство решает не только сколько давать еды и бабла
чо?

>в макакокодеры + по вечерам формулки дрочить как хобби

лол, как что-то плохое. если попасть в сша и там в макакокодеры можно и формулки не дрочить. кодерам там деньги платят, в отличие от математиков, особенно здесь.

я к тому что математика и должна быть хобби. как и искусство например. музыканты должны иметь дневную работу. математики должны иметь дневную работу. хоть кодерами хоть кочегарами. это очевидный факт
Аноним 17/05/16 Втр 05:09:37  363327
>>363326
>математика и должна быть хобби
Так кто кому запрещает дрочить формулы если ему так нравится. Речь же идёт про "мне обязаны деньги платить потому что я очень нужен!".
Аноним 17/05/16 Втр 05:11:10  363328
>>363327
сори не разобрался. с этим я согласен конечно
Аноним 17/05/16 Втр 05:47:11  363330
>>363281
Я тоже за.
Аноним 17/05/16 Втр 05:58:43  363333
>>363075
Александров, Лекции по аналитической геометрии.
Погорелов, Аналитическая геометрия.
Аноним 17/05/16 Втр 06:02:15  363334
>>363333
Спасибо, анон.
Аноним 17/05/16 Втр 06:50:40  363336
https://www.youtube.com/watch?v=tffkICJjfqQ
Аноним 17/05/16 Втр 07:01:17  363338
https://www.youtube.com/watch?v=IU6zuPi2Epk
Аноним 17/05/16 Втр 07:53:10  363350
14634607909450.png (16Кб, 517x539)
>>363338
Лол. Таблица производных.
Аноним 17/05/16 Втр 07:58:09  363351
>>363338
Суть видео: производная функции показывает скорость изменения y от x, у первообразной функции.
Аноним 17/05/16 Втр 08:20:46  363353
>>363338
Батя говорит, физика - царица наук, а математика фуфло для распила бабла. Это так?
Аноним 17/05/16 Втр 08:27:52  363355
>>363353
Философия - царица наук. Не зря в нормальном мире доктора наук = доктора философии.
Аноним 17/05/16 Втр 08:31:46  363356
>>363355
>Философия - царица наук
>Философия
>наук
А маску Гая Фокса ты не забыл, феласов?
Аноним 17/05/16 Втр 08:41:23  363357
>>363356
Батенькин сынок плиз.
Аноним 17/05/16 Втр 08:43:36  363358
>>363357
ТЫ НЕ ГОНИ БАТЯ УМЁН НЕ ГОДАМ!
Аноним 17/05/16 Втр 12:50:10  363384
>>363317
КО КО КО ГОСУДАРСТВО СТРОНГ КО КО КО ОНО ТЕБЯ РАЗМАЖЕТ КО КО КО СИДИ И НЕ РЫПАЙСЯ КО КО КО

Маня, я где-то предлагал свергать государство или требовать особых прав для математиков?

Я выложил трёп авторитарных совков, которые даже на НАУЧНОМ, блять, ФОРУМЕ в ответ на обсуждение программы математического факультета начинают кукарекать про то, что надо вообще запретить математику блядскую и давать экономику с программированием и физикой, чтобы суки вместо гомологий своих учились преподавать в вузах для других специальностей.

То есть ещё раз: обсуждают как улучшить математическую программу на факультете, где учат математику, она и так не идеальна.
Приходят авторитарные совки и предлагают не улучшить, а ухудшить, чтобы КО КО КО государственный деньги не впустую шли.

И это на научном форуме.
Аноним 17/05/16 Втр 12:53:47  363386
>>363327
>>363323

Смотри
>>363384

А под "нужно ли государство математике" подразумевалось, конечно же, нужно ли "ТАКОЕ государство"



Аноним 17/05/16 Втр 14:09:54  363394
>>363336
Вопрос "часто людьми поиска становятся..." как будто Рома Михайлов составлял.
Аноним 17/05/16 Втр 15:12:41  363396
>>363326
>я к тому что математика и должна быть хобби.
Тогда эта область науки умрет. С очень печальными итогами для всей точно-технической науки, лет так через 10-100.
>музыканты должны иметь дневную работу. математики должны иметь дневную работу.
У музыкантов как раз есть дневная работа(ы), связанная непосредственно с музыкой. Если они, конечно, профессиональные музыканты. Музыкант который с 9 до 18 настраивает инструменты по вызову, а вечерком бренькает на гитарке - не музыкант, а настройщик. Если он учит детвору в муз. школе - он не музыкант, а преподаватель. Есть же проф. музыканты. Играют на заказ, играют в оркестрах, сдаются в аренду и т.п. и т.д. Полная занятость, частичная занятость, все как у людей. И это не хобби, это именно первичный источник доходов. Есть проф. писатели, которые писанием макулатуры зарабатывают на жизнь, есть те для которых графомания - хобби или дополнительный источник бабла. Аналогично и в математике, да и в любой профессии. Разве что в науке те кто решают (читай государства) тверду убеждены что ученый обязан быть одновременно еще и преподом. Защкварищще.

>>363384
Научный, блджад форум. Рассмешил. Но давай по порядку. Пробегся, две странички почитал. Вес той дискуссии примерно равен весу треда на дваче. Но проблема есть: чистые математики нынче не востребованы настолько, что бы иметь отдельные факультеты, цель которых - клепать чистых математиков. И если дипломник физик еще и способен как-то адаптироваться где-то еще, то чистый математик, похоже - нет. Слишком мозги едут, наверное. Соответсвенно, государству не выгодно иметь выпускников без работы (их, правда, мало, так что государству похуй) и не выгодно кормить факультеты где клепают ненужные кадры (тут уже бабла побольше, не похуй).
>>363386
>"ТАКОЕ государство"
Не нужно, но лучше государства, я думаю, ты не найдешь. Математика везде в упадке (читай нахуй не нужна). Да и точные науки вообще - переполнены кадрами. В России в этом смысле ситуация гораздо лучше: работу в науке найти гораааааздо легче, за счет нищенской зарплаты.
Аноним 17/05/16 Втр 15:23:08  363401
>>363208
Вообще-то имелись в виду все книги из ОП, и обновленной программы Вербицкого.
Аноним 17/05/16 Втр 15:51:06  363407
>>363396
Ну пример с музыкантами совершенно неудачный, игра на музыкальных инструментах это ведь готовый продукт. А вот композиторство, другое дело.
Аноним 17/05/16 Втр 15:53:07  363408
>>363396
>Научный, блджад форум. Рассмешил. Но давай по порядку. Пробегся, две странички почитал. Вес той дискуссии примерно равен весу треда на дваче. Но проблема есть: чистые математики нынче не востребованы настолько, что бы иметь отдельные факультеты, цель которых - клепать чистых математиков. И если дипломник физик еще и способен как-то адаптироваться где-то еще, то чистый математик, похоже - нет. Слишком мозги едут, наверное. Соответсвенно, государству не выгодно иметь выпускников без работы (их, правда, мало, так что государству похуй) и не выгодно кормить факультеты где клепают ненужные кадры (тут уже бабла побольше, не похуй).

Университет, всё же, должен давать высшее образование (со всеми плюсами и минусами этого самого образования), а не быть курсами профессиональной подготовки. Если государству нужно больше кадров - пусть делает больше техникумов и меньше университетов, но университеты пусть делает именно университетами.
Аноним 17/05/16 Втр 16:48:07  363416
>>363407
Аноны, у меня дрим такой - всем думающим и талантливам людям переехать в америку. Америка это страна управленцев, белых воротников. Азиаты пусть ебошат на фабриках, не вижу в этом моральных мук. А нам нужно всем работать в офисах. Или в общепите накраяйняк. Но это day time job. А вечером - пожалуйста, хоть музыку пиши хоть формулы, хоть хуем груши. Вот лейбниц кто был? Клерк обычный, хуйней страдал. Математика у него хобби была. И в истории много математиков-любителей который были близки к успеху.
Аноним 17/05/16 Втр 16:51:40  363417
>>363396
>>363396
Кстати пейсатель - это 100% хобби должно быть. Еще бы придумали профессиональных поэтов. Это ж совковый пафос - Дом, ебта, пейсателей, с Рестораном в Который не всех пускают. А в новом эффективном мире они должны на заводе ебошить или как-то по другому пользу приносить. Самоиздаваться теперь легко.
Аноним 17/05/16 Втр 17:09:40  363418
>>363407
Дак наука тоже в основном игра на готовых инструментах. Даже математика.
Понятное дело, что один в один соответствие не будет, но общая структура везде схожа.

>>363408
Это немного не вписывается в международную систему, где есть универы и больше ничего. А не универы, институты (не НИИ), техникумы хуехникумы и прочее говно. Россиянская система решила следовать усредненной международной, со всеми вытекающими.

>>363416
>Вот лейбниц кто был? Клерк обычный, хуйней страдал.
Не те времена нынче. Любителям в науке места нет. Слишком все дорого, слишком большой объем. Все простые вещи уже придуманы и открыты, а теоретические философствования выпадают из моды. Натеоретизировали уже на 10 лет вперед.

>>363417
Писательство - это не только худ. литература вроде книжек-романов. Это еще и всякая тех. писанина, и прочая писанина на заказ, вроде сценаристов для голивудиков. Но то что проф. худ. лит. скоро исчезнет - я тоже так думаю. Прибыльность - тю-тю, интернетик-либрусечик, выживут уже заранее разжиревшие.
Аноним 17/05/16 Втр 17:13:49  363419
>>363418
Труд музыканта, играющего на публику, сравним с ремеслом инженера, нежели математика.
Аноним 17/05/16 Втр 17:26:31  363422
>>363418
>Это немного не вписывается в международную систему, где есть универы и больше ничего. А не универы, институты (не НИИ), техникумы хуехникумы и прочее говно. Россиянская система решила следовать усредненной международной, со всеми вытекающими.
Неправда, в США есть колледжи и есть университеты, например, и человеку, зачастую, незашквар закончить (только) колледж, а не университет.
Аноним 17/05/16 Втр 17:30:04  363423
>Не те времена нынче. Любителям в науке места нет. Слишком все дорого, слишком большой объем. Все простые вещи уже придуманы и открыты, а теоретические философствования выпадают из моды. Натеоретизировали уже на 10 лет вперед.
Слишком субъективно, на мой взгляд, как первое так и второе утверждение.
Аноним 17/05/16 Втр 17:40:38  363425
>>363419
Ах если бы.. Но сейчас в моде "кококо я жи нологоплотельщик, а ну ка отчитывайтесь, на что я свои драгоценные денюшки плачу, да и еще так что бы я понял???". Вот и играют все в той или иной мере на публику. Как минимум вне бСССР. Одна из причин почему туоретикам и математикам непросто.

Но это все не по теме треда, прощу прощение за такое отклонение от сути. Хотелось бы высказать огромное спасибо анону >>362794. Я это имя пять лет вспомнить пытался (не целенаправленно, но в голове вопрос зудел).
Схожий вопрос: никто не подскажет задачник по диф.урам? Диф. и инт уравнения, относительно тоненький такой. Перед каждым разделом было хорошее, но краткое вступление с описанием методов (что конечно стандарт для задачников, но этот почему-то запомнился особо). Вроде бы книжечка коричневая была, но память может врать. наверняка стандарт в любом себя уважающем вузе со всякими физ и мат факультетами.
Аноним 17/05/16 Втр 17:52:57  363427
>учит детвору в муз. школе - он не музыкант, а преподаватель
У меня есть стойкое ощущение, что 99% музыкантов во всем мире на хлеб зарабатывают именно преподаванием. А профы - туторством. И тд. Пирамида ведь.


>наука тоже в основном игра на готовых инструментах
>Любителям в науке места нет

Если обобщить все эти мысли из разных комментов, то все примерно об одном и том же. Есть некая "илита", пейсатели, музыканты, ученые, проф. спортсмены которые уже arrived. Очень мало их. Остальным не светит. Сейчас entry point во все сферы науки и творчества очень низкий. Только захотеть. Все на гугле есть. Отсюда высокая конкуренция и отстуствие профита. Технарям если смириться с мыслью что ты ремесленник - погромист, ученый, и тд то можно хорошо жить и протирать штаны в офисе, а вечером хоть формулы дрочи хоть музыку пиши. А вот чисто людям искусства нужно искать дневную работу.




Аноним 17/05/16 Втр 18:31:43  363443
Если ученый из страны X получает какие-то важные результаты, престиж страны X растет. Из таких же соображений государство вливает деньги, например, в спорт. То есть, спрашивать "зачем государству платить за гиперкэлеровы многообразия" это все равно, что спрашивать "зачем государству проводить олимпиаду в Сочи или чемпионат по футболу". На спорт по понятным причинам денег тратится намного больше чем на науку, но суть та же.
Аноним 17/05/16 Втр 18:43:28  363445
>>363443
>зачем государству проводить олимпиаду в Сочи
1933
распил
триумф перед войной в украшке
Аноним 17/05/16 Втр 19:47:54  363453
>>363425
Филлипов?
Аноним 17/05/16 Втр 19:51:51  363456
>>363443
Слишком много денег - слишком мало престижа. Футбол и прочий спорт, кстати, это уже скорее прерогатива бизнеса, а не государства, потому что на футбольных матчах можно пивко рекламировать. А что можно рекламировать во всякой писанине на архиве?
Аноним 17/05/16 Втр 21:08:53  363465
>>363456
> Слишком много денег - слишком мало престижа.
Каких денег? Нужно давать математику минимум денег, чтоб он от голода не умер. Большинство математиков полнейшие аутисты и если им предложат заниматься математикой фулл тайм за еду, они согласятся. Я бы согласился.

И ты недооцениваешь их влияние на другие сферы. Люди, которые работают в индустрии но занимаются какой-то околоисследовательской деятельностью (возьмем, например, Яндекс) много общаются с математиками, учатся у них. Скажем, курсы в ШАД читают Райгородский, Ширяев и другие. А они тоже не из ниоткуда взялись, они выросли в какой-то математической среде. В стране, в которой математической среды не будет, наступит полнейшая деградация, не будет вообще никакой научной и близкой к научной деятельности. Все нормальные люди, хоть чуть-чуть нуждающиеся в общении, будут съебывать и элементарно некому будет учить тех же инженеров. А даже сырьевому придатку нужны инженеры для обслуживания нефтяных вышек.
Аноним 17/05/16 Втр 21:16:56  363467
Ребята, есть один эллипс и его уравнение: 16х^2+9y^2=144. И я ну никак не могу понять, как мне так изъебнуться, чтобы по этому уравнению вычислить его объём по оси х через опр интеграл, с пред -3 и 3. Прост как я не выражаю х и у - получается полная ебала 0. Буду очень признателен. Уже, блядь, давно посчитал по обычной формуле, но преподше похуй, ей нужен интеграл.
Аноним 17/05/16 Втр 21:40:45  363480
Охуеть. Решаю задачи к элементарной книжечке для школьников. Первая задача, показать иррациональность числа sqrt(2). Следующая за ней - ДОКАЗАТЬ ТЕОРЕМУ ДЕДЕКИНДА О СЕЧЕНИЯХ, сука блдь.
Аноним 17/05/16 Втр 21:51:02  363487
Ко-ко-ко математики не нужны ко-ко-ко государственные деньги ко-ко-ко мы налогоплательщики
Идите нахуй из треда, тролли ебаный
И да, на математическом факультете должны учить математике точно так же, как и на философском - философии, на филологическом - филологии, на экономическом - экономике, на физическом - физике
Университет - это не рабфак и не техникум, это место, где дают высшее образование, учат наукам.
Все инакомыслящие могут сразу пойти нахуй
Аноним 17/05/16 Втр 21:51:13  363488
>>363467
Господи, какой же еблан... Разобрался.
Аноним 17/05/16 Втр 21:52:01  363489
>>363487
Двачую этому. Вышка нужна только для научной деятельности.
Аноним 17/05/16 Втр 21:55:59  363492
>>363480
Да уж, "доказать" это громко звучит. По сути надо не забыть формальные проверки, что сечение является сечением, что число является наибольшим или наименьшим в одном из классов, что одновременно один из вариантов быть не может.
Аноним 17/05/16 Втр 22:02:51  363495
>>363492
До этого в книге говорили только что такое открытое множество и стягивающаяся последовательность.
Аноним 17/05/16 Втр 22:04:06  363496
>>363495
Скрин книги вкинь.
Задачи и немного материала до неё.
Аноним 17/05/16 Втр 22:05:42  363497
>>363480
>ДОКАЗАТЬ ТЕОРЕМУ ДЕДЕКИНДА О СЕЧЕНИЯХ
Ну короч я в аксиоматическом построении R работаю, так что следует из аксиомы непрерывности.
Аноним 17/05/16 Втр 22:12:59  363499
14635123790630.png (41Кб, 395x500)
14635123790631.png (66Кб, 425x559)
14635123790642.png (33Кб, 395x398)
>>363496
Пара рандомных страниц и задачи после них.
Аноним 17/05/16 Втр 22:13:45  363501
>>363487
Школьник, ты заебал уже кудахтать в треде с разорванным пердаком о маняфантазиях насчёт собственной нужности.
Аноним 17/05/16 Втр 22:20:29  363506
>>363501
Маня, где я(или кто-то ещё) писал о своей нужности?

Но раз уж ты завёл этот разговор, то отвечаю. Математикам(как и всем нормальным людям) как-то похуй, нужны ли они таким пидорам, как ты. Вы им-то вряд ли нужны(хотя хз, мб ты какой-нибудь мегаврач, который дохуя жизней спасает, но я сомневаюсь, больше похож на пидора выпускного возраста с юношеским максимализмом).

Аноним 17/05/16 Втр 22:23:39  363508
Пацаны, поясните за сечения Дедекинда, зачем они нужны? Я думал R определяется как классы эквивалентности фундаментальных последовательностей. Это же очень естественный подход.
Аноним 17/05/16 Втр 22:27:25  363509
14635132453300.png (101Кб, 555x639)
14635132453311.png (194Кб, 1202x899)
14635132453322.png (165Кб, 1206x908)
14635132453343.png (124Кб, 580x828)
>>363495
Так для доказательства теоремы Дедекинда нужен более глубокий теоретический материал, подкину тебе чутка.
Аноним 17/05/16 Втр 22:32:39  363511
14635135593330.jpg (57Кб, 538x305)
>>363508
>Это же очень естественный подход.
Вербит говорит что не очень.
Аноним 17/05/16 Втр 22:35:32  363513
>>363511
Хмм, тогда да.
Вербицкий > хуй-пидор из поста >>363508
Аноним 17/05/16 Втр 22:43:12  363517
>>363509
А что за книжка? Для ньюфага сложно будет?
Аноним 17/05/16 Втр 22:55:36  363522
Нашёл у себя Фихтенгольца Г.М. Основы математического анализа 1. Можно с него начинать, если все уже забыл?
Аноним 17/05/16 Втр 23:04:51  363524
>>363506
Вот давай школьник пойдёт нахуй и не будет от имени "математиков" что-то, охуевший совершенно, говорить.
Аноним 17/05/16 Втр 23:12:40  363526
>>363524
Давай ты просто пойдёшь нахуй. Иди в какой-нибудь ватный тред и там обсуждай "государство" и "что ему нужно".
А к нормальным людям не лезь, уебок.
Аноним 17/05/16 Втр 23:21:43  363529
>>363511
> определение получается очень неявным
Ну это субъективно. По-моему эта идея несложная и естественная. Ну да, классы эквивалентности - это, может быть, немного абстрактно. Но я думаю, если человека интересует определение R, он уже привык к какому-то минимальному уровню абстракций.
Аноним 17/05/16 Втр 23:24:49  363530
>>363396
>Научный, блджад форум
А какой?
>Вес той дискуссии примерно равен весу треда на дваче
Поржать-то над ними можно. Собрались совки и обсуждают хуйню. Задали вопрос "как улучшить матобразование", ответ же "как его ухудшить, чтобы бедное государство не тратило деньги просто так"(ага, обучение 60 человек - это пиздец как много на фоне всеобщего пиздеца, воровства, распилов бабла, и траты over9000 % бюджета на ебаную оборонку, чтобы рептилоиды нас не захватили)
>Но проблема есть: чистые математики нынче не востребованы настолько, что бы иметь отдельные факультеты, цель которых - клепать чистых математиков.

Какой-то однобокий взгляд. Кто тебе вообще сказал, что цель какого-то факультета - это кого-то клепать? Факультет даёт вышку, а дальше сам. Хочешь - иди сосать хуи за деньги, хочешь - иди в банк, хочешь - занимайся наукой, хочешь - нихуя не делай.

> И если дипломник физик еще и способен как-то адаптироваться где-то еще, то чистый математик, похоже - нет. Слишком мозги едут, наверное.

Да, авторитетное мнение, очень адекватное.
Погугли, сколько математиков в финансах, хеджевых фондах, сколько их основали, сколько занимаются алгоритмической торговлей и т.д.

Но причём тут это? Это всё не относится к матобразование(как и к физобразованию)

Хуле ты забыл в треде с такими заявлениями про "мозги едут"? Пошёл нахуй.

>Соответсвенно, государству не выгодно иметь выпускников без работы

Похуй на государство.


> не выгодно кормить факультеты где клепают ненужные кадры

Как-то похуй, что им выгодно.

Сегодня они не будут финансировать фундаментальную науку, а завтра вернёмся в пещерный век.

>Математика везде в упадке

Нихуя. Полный бред человека не знакомого с сабжем. Да, ученым не дают дохуя бабла просто так, но никто и не требует. Это не называется "в упадке". А в Рашке пиздец, да. Но и тут кто хочет, занимается наукой и не ноёт.


Аноним 17/05/16 Втр 23:26:39  363531
>>363526
Таки я-то к нормальным людям и не лезу. Более того, я вообще ни к каким людям не лезу, это ты тут на весь тред рвёшься и пристаёшь ко всем со своим "ничо ни должин".
Аноним 17/05/16 Втр 23:38:47  363532
>>363308
Банп вопросу.
Аноним 17/05/16 Втр 23:54:33  363533
http://ebook3000.com/Inventing-the-Mathematician--Gender--Race--and-Our-Cultural-Understanding-of-Mathematics_356896.html

Зацените
Книга про то, что "белые хуемрази угнетатели" специально придумали такую математику, которая непонятна "богиням" и "цветным"

Охуенчик
Аноним 17/05/16 Втр 23:55:43  363534
>>363533
Нормас.
Аноним 17/05/16 Втр 23:56:40  363535
>>363396
>Разве что в науке те кто решают (читай государства) тверду убеждены что ученый обязан быть одновременно еще и преподом. Защкварищще.

Миша Вербицкий считает, что учёный, который не преподаёт, это гнида.
Аноним 18/05/16 Срд 00:03:00  363536
>>363533
В описании про угнетение ничего нет. Ты читал эту книгу или просто додумал?
Аноним 18/05/16 Срд 00:05:57  363537
14635191576280.jpg (38Кб, 486x614)
>>363535
по методическим пособиям, я полагаю. по программе одобренной этим государством, с не обходимостью вторую половину времени тратить на заполнение отчёта о преподавании по методическим пособиям выверенным по программе специально одобренной этим государством
все знания мира в интернете есть, изучай. нет, хочу слушать бубнёж поехавшего (лекции похожи на телевизор, который похож на свиное корыто, т.е. жри что дают и не выбирай то что нравится
Аноним 18/05/16 Срд 00:06:21  363538
>>363517
ландау, основы анализа. Теорема о сечениях это самое сложное, что в ней есть.
Аноним 18/05/16 Срд 00:08:47  363541
>>363538
Ландау это как Ландау-Лифшиц?
Аноним 18/05/16 Срд 00:13:18  363542
>>363541
Это как открываешь книгу и читаешь.
Аноним 18/05/16 Срд 00:14:18  363543
>>363537
Это лекции у себя на диване, а у препода можно спросить че-нибудь или подойти после к нему. Вот это самое ценное, экономит тяжелые раздумья "не понимаю нихера".
Аноним 18/05/16 Срд 00:18:01  363545
>>363543
Хотел бы просвещать - оставил бы асечку-писечку студентоте. Сколько лекций я за свою жизнь исписал - помню только сплошь хуету какую-то.
Причём, не факт что препод знает. При том что выбирать их не повыбираешь (за всю жизнь лишь одного препода сдвинули, и то лишь потому что он с дурки и грязно домогался до парней)
Аноним 18/05/16 Срд 00:20:43  363546
>>363536
ОФК, я всю прочитал
Ведь надо сожрать всё собачье дерьмо на улице, чтобы понять, что наступать в него не стоит
Аноним 18/05/16 Срд 00:22:27  363547
>>363542
Ну вот че ты, ну вот че ты? Я, например, никаких Ландау, кроме Льва, не знаю. Как этот учебник хоть, по сравнению с анализом Тао?
Аноним 18/05/16 Срд 00:24:20  363548
>>363546
Может годная книга. Переосмыслишь свою жизнь, станешь веганом, книги оранжевые читать начнешь, в попу дрюкаться.
Аноним 18/05/16 Срд 00:48:42  363549
>>363547
Он нечитабелен.
Нравится Тао? Читай дальше его.
Более краткий вариант(как Рудин, но читабельнее) - Pugh "Real Mathematical Analysis"

Там тоже используются Дедекиндовы сечения, но не очень подробно и свойства не проверяются, они чисто используются как пример и для док-ва свойства полноты.
Аноним 18/05/16 Срд 00:50:11  363550
>>363547
Кстати, я бы на твоём месте не парился, как определяются вещественные числа - сечениями или классами эквивалентности фундаментальных последовательностей.

Ну, определили и определили. Дальше пойдём.
Аноним 18/05/16 Срд 00:50:34  363551
>>363281
Было бы круто, но как конкретно ты себе это представляешь?
Аноним 18/05/16 Срд 00:54:35  363552
Давайте составим рейтинг матвузов.
Есть матфак ВШЭ, мехматы МГУ и НГУ, матмех СПБГУ, ФУПМ МФТИ

Че круче, че хуже, че говно, че божественно?
Аноним 18/05/16 Срд 02:31:29  363554
>>363552
Тянки универсальные в каких? Чтобы
1) им нравились задроты 4/10 потому что они умные (эквивалент 7/10)
2) чтобы сами они были >7/10 внешне
3) и >4/10 мозгами

В каких матвузах они отвечают этим критериям?
Аноним 18/05/16 Срд 05:45:26  363556
>>363453
ДААААА, сука, как же тяжело жить с дырявой головой. Спасибо.

>>363530
>Нихуя. Полный бред человека не знакомого с сабжем.
Хорошо, допустим я ошибаюсь. Математика не в упадке. Теперь мы оба идем на, например, сайт CODRISа, смотрим проекты FP7. Математика и статистика - 239 проектов за все семь лет. Сравниваем с, например, физика - 1900, сравниваем с Scientific Research - 10200. Мизер, но не так уж все плохо? 200 преоктов, это считай (в среднем примерно) наверняка больше миллиарда евро!! А теперь смотрим на мат проекты и ищем хотя бы один, который только в категории математика. А не "математика" и "физика", математика и химия и т.п. Ноль на первой странице, ноль на второй, а дальше меня заебало. Потому что мне и так известно, что математика - мертва, и единственный шанс математику вести исследование и получать гранты - подсосаться к физикам, биологам или хотя бы всяким social sciences.
На остальное отвечу потом. Рабочее время таки.
Аноним 18/05/16 Срд 06:33:50  363558
>>363556
Оценивать состояние математического знания по количеству денег в неё вливаемую - это лол полный. Позволь спросить, это в какой же период времени в неё вливалось много денег? В ХХ веке, хочешь сказать, ситуация была кардинально иной? Или математика мёртвая с рождения, по твоему мнению?
Аноним 18/05/16 Срд 08:08:25  363563
>>363558
Не математического знания, а математики как области науки. В XX веке ситуация была иной, так как наука была иной и мир не такой. Бабла было много, мест рабочих было много и наука цвела. Вливания извне были не критичны. Сейчас же, зарплаты в первом мире не постоянным сотрудникам (не профессорам), т.е. постдокам, research fellows и прочим контрантникам (в том числе иногда и ПхД студентам) идут на 99% из вливаемых извне денег, а не из вузика. Контрактники - уже обязательный этап в карере ученого, так как позицию профессора тебе не даст никто сразу после ПхД. То есть ситуация простая: окончив вуз/аспирантуру, ты остаешься в математике-науке только если подсасываешься к другим наукам и бартачишь на них. Вливание в чистую математику нынче примерно навно нулю (не "мало", а именно "ничего").
Конечно, присасывание бывает чисто формальным, на уровне мотивации "почему моя абстрактная хуйня может быть полезна для нормальной науки". Без таких послаблений математики бы уже вымерли как вид, что понимают ученые, но не могут понять те у кого в руках рычаги управления и ключи от сейфа с баблом.
Аноним 18/05/16 Срд 09:01:42  363565
>>363563
https://www.mathjobs.org/jobs?joblist-0-0----
>То есть ситуация простая: окончив вуз/аспирантуру, ты остаешься в математике-науке только если подсасываешься к другим наукам и бартачишь на них. Вливание в чистую математику нынче примерно навно нулю (не "мало", а именно "ничего").
Нажал ctrl+F написал "postdoc" нашло куча "чистых" позиций. ЧЯДНТ?

Аноним 18/05/16 Срд 09:53:47  363574
>>362636
Нахуй так много?
Аноним 18/05/16 Срд 10:55:34  363581
>>363565
Не читаешь описания позиций. Если бы читал - то увидел бы, что большиство "чистых" оказывыаются Computer science, “Developing Mathematical Tools for Complex Materials Systems,” "Computational Modeling of Cancer", "Mathematical Oncology" и подобное.
Вот тебе чистая математика, судя по всему: https://www.mathjobs.org/jobs/jobs/8567
Еще чистых нашел - два Китай и одна Корея с Тайванью (все еще научно развивающиеся страны, там с работой легче).
При чем как я писал выше - присасывание бывает чисто формальным. Главное уметь хитро изъебнуться перед теми кто с деньгами, а потом уже с деньгами искать людей на чистые задачи. Грантопильство высшей категории.
Аноним 18/05/16 Срд 11:25:39  363583
>>363581
> "Computational Modeling of Cancer", "Mathematical Oncology"
Охуенчик было бы с раком побороться. Тема вообще благородная, не то что ваша алгебраическая геометрия ссаная.
Аноним 18/05/16 Срд 11:49:07  363593
Скоро сдавать ЦТ, математику знаю на уровне 9 класса. Посоветуйте годноты, чтобы выучить за месяц более-менее нормально.
Аноним 18/05/16 Срд 12:05:03  363595
>>363593
Рассел Б., Уайтхед А. - Основания математики. Трехтомник.
Аноним 18/05/16 Срд 12:34:43  363607
>>363554

Нахуя тебе тянки, блять? Если есть математика.

Какие тянки, тян не нужны же(куны тоже). Математику иди изучай. Епт.
Аноним 18/05/16 Срд 13:24:11  363617
>>363593
Зорич.
Аноним 18/05/16 Срд 13:24:30  363618
>>363617
Или Рудина.
Аноним 18/05/16 Срд 13:29:16  363621
>>363583
На безрыбье и рак щука. Вон мамфорд тоже например забил на это говно ненужное, теперь занимается моделями для зрения. Жизнь такая.
Аноним 18/05/16 Срд 13:36:18  363622
14635677789100.jpg (45Кб, 600x405)
>>363621
Он красивый.
Аноним 18/05/16 Срд 13:40:25  363624
>>363547
Этот "учебник" читается за 2-3 часа, так что тебе самому решать.
Аноним 18/05/16 Срд 13:42:08  363625
14635681289850.jpg (77Кб, 720x480)
>>363583
>Охуенчик было бы с раком побороться.
За раком стоит краб не только путин, вся эта система сплошной рак
https://www.youtube.com/watch?v=s3QkjzypXEI
https://www.youtube.com/watch?v=RwZW94KPce8
Аноним 18/05/16 Срд 15:00:42  363636
>>363583
> Охуенчик было бы с раком побороться
Выкупи двач у Абу.
Аноним 18/05/16 Срд 15:36:18  363640
Проиграл с зацензуренного петушка.
Аноним 18/05/16 Срд 16:00:17  363641
А по курсу, который шабат сейчас в НМУ читает и первому полугодию будут записки лекций, никто не знает?
Аноним 18/05/16 Срд 16:10:23  363642
>>363641
Нах?
По нему же не выучишь алгебру. Это же какая-то жуть - смесь фактов алгебры без доказательств и теории категорий/гомологической алгебры.

Лучше взять нормальный учебник по алгебре и читать, не?
Аноним 18/05/16 Срд 16:58:44  363645
>>363593
Мне бы твои проблемы. Мне скоро сдавать ЦП. Посоветуйте годноты.
Аноним 18/05/16 Срд 16:59:31  363646
>>363645
Рыбников?
Аноним 18/05/16 Срд 17:05:00  363648
>>363642
Но мне не нужно учить алгебру, я хочу посмотреть что там.
Аноним 18/05/16 Срд 17:07:30  363649
14635804503650.png (2Кб, 233x38)
Как это начертить?
Аноним 18/05/16 Срд 17:08:27  363650
>>363649
Карандашом. Ну или ручкой.
Аноним 18/05/16 Срд 17:09:40  363652
>>363650
Не могу.
Аноним 18/05/16 Срд 17:11:27  363653
>>363650
ПОЛУЧИЛОСЬ!
Аноним 18/05/16 Срд 17:19:14  363655
>>363649
Раскрыть скобки, выделить полный квадрат, получится уравнение окружности после смещения начала координат.
Аноним 18/05/16 Срд 17:20:19  363656
>>363655
>получится уравнение окружности после смещения начала координат
да, уже понял, лол. Но всё равно спасибо.
Аноним 18/05/16 Срд 18:07:52  363665
>>363574
Коллекционер
Аноним 18/05/16 Срд 18:16:57  363667
>>363583
рост туморов моделируется кажется error function. кроме того похожие сигмоиды возникают например в радиологии. например adjuvant treatment после мастектомии. идея в том чтобы заебошить возможно оставшиеся раковые клетки. при этом хорошо бы не положить дохуя своих. и там как раз есть therapeutic window в виде двух сигмоидов и нужно попасть примерно посередке. математика и лечит рак хуле.
Аноним 18/05/16 Срд 19:13:50  363670
>>363583
Есть интересные работы по фрактальной размерности раковых опухолей. Есть надежда, что это признак отличия любой опухоли от любой здоровой ткани, просто как два разных класса объектов. М.б. это будущее диагностики рака, а занимаешься по факту фракталами, прикинь?
Аноним 18/05/16 Срд 19:17:07  363671
Я полиглот. Какой язык вам выучить?
Аноним 18/05/16 Срд 19:18:17  363672
>>363671
Coq.
Аноним 18/05/16 Срд 19:18:46  363673
>>363672
Хкита без задач.
Аноним 18/05/16 Срд 19:21:16  363674
>>363673
lol
Аноним 18/05/16 Срд 19:25:18  363676
Пацаны, как вы считаете, можно ли каким-то образом использовать машинное обучение, чтоб комп с помощью Coq доказывал простейшие свойства математических объектов?
Аноним 18/05/16 Срд 19:27:28  363677
>>363676
)
Аноним 18/05/16 Срд 23:07:42  363705
Парни есть факультет алгебраической геометрии в Москве или Питере?
Желательно чтобы баллы невысокие были.

Аноним 18/05/16 Срд 23:54:07  363708
14636048480180.png (125Кб, 736x736)
>>363705
>есть факультет алгебраической геометрии
Аноним 19/05/16 Чтв 00:16:02  363712
>>363705
есть кафедра во ВШЭ, но она для быдловербиток и карьеристов, если хочешь заниматься математикой туда идти нельзя.
Аноним 19/05/16 Чтв 00:19:25  363714
Епт, аноны, объясните мне понятие компактности. Только чтобы совсем как детсадовцу, там как один зайчик плюс еще один зайчик равно два зайчика.
Аноним 19/05/16 Чтв 00:23:26  363715
>>363714
Как одного зайчика от другого отличаешь?
Аноним 19/05/16 Чтв 00:25:26  363717
>>363715
Морфологически.
Аноним 19/05/16 Чтв 00:29:08  363719
>>363717
Ну вот два совершенно одинаковых зайчика. Почему же они не сливаются в одного?
Аноним 19/05/16 Чтв 00:33:11  363723
>>363719
Я чет даже без пол-литра не отвечу. Они же отделены друг от друга.
Аноним 19/05/16 Чтв 00:41:08  363725
>>363723
Отож. Отделены они своей компактностью. Каждый зайчик где-то кончается.
А вот взять родословную этих же зайчиков. Допустим, появилась у тебя способность видеть родственников этих зайчиков, из прошлого и будущего, и вносить эти данные в единое генеалогическое древо. Вот оно компактным не будет, зайчики станут связаны общим предком из прошлого и наплодят бесконечно потомков в будущем
Аноним 19/05/16 Чтв 00:45:54  363726
>>363725
А зачем для этого требовать компактность зайчиков? Разве не достаточно потребовать их ограниченность?
Аноним 19/05/16 Чтв 00:51:54  363728
>>363726
Детсадовцы не должны такое спрашивать
Аноним 19/05/16 Чтв 01:13:45  363730
14636096255020.png (47Кб, 381x512)
>>363728
Я, собственно, почему про все это спросил-то. Почему пересечение икс и у дает открытые множества, и почему их объединение равно икс, а не икс минус пересечение икс и окрестности игрек?
Аноним 19/05/16 Чтв 09:09:42  363752
>>363730
Опять ты с этой книгой? Я же тебе говорил, возьми нормальную, не мучайся так.

Википедия говорит, что
> Компа́ктное простра́нство — определённый тип топологических пространств, обобщающий свойства ограниченности и замкнутости в евклидовых пространствах на произвольные топологические пространства.

Ну и все как бы. Че еще надо?

Вникать в этот понос на пикрелейтед никто не будет, потому что, во-первых, подход в этой книге не является общепринятым, во-вторых, ты даже не написал, что за теорема и что доказывается, что за x и y.

Учи топологию нормально: сначала определяется топологическое пространство, потом метрическое; определяется компактность в общем смысле, а потом доказывается, что в R^n компактное множество = ограниченное и замкнутое множество.
Аноним 19/05/16 Чтв 09:50:19  363755
>>363714
Пространство компактно, если из всякого его покрытия открытыми множествами можно выбрать конечное подпокрытие.

>>363726
Зайчики могут отказаться неотделимыми.
Аноним 19/05/16 Чтв 10:18:24  363764
>>363752
Забыл определение написать, но этот >>363755 анон написал.
Аноним 19/05/16 Чтв 11:01:03  363776
Изучающие матан по Тао: не подскажете, где нашли? Потратил часа полтора поисков, оббежал несколько трекеров, не нашел.
Аноним 19/05/16 Чтв 11:18:08  363778
>>363776
>матан
Аноним 19/05/16 Чтв 12:17:45  363787
>>363776
В оппосте снизу написано.
Аноним 19/05/16 Чтв 12:56:27  363791
>>363787
Пиздоглазый я, извиняюсь. Слона не увидел.
Аноним 19/05/16 Чтв 18:31:36  363871
Есть ли смысл, если у меня IQ 80?
Аноним 19/05/16 Чтв 18:45:16  363872
>>363871
Да. Только начинай по порядку.
Аноним 19/05/16 Чтв 21:51:40  363890
>>363712
Почему туда не стоит идти?
Аноним 19/05/16 Чтв 22:00:55  363892
>>363676

Я вот собираюсь этим заниматься в качестве хобби. Обрати внимание на homotopy type theory - очень объемная тема, с кучей вариаций, можно завязнуть, но
есть вот такая штука https://github.com/HoTT/HoTT - формализация этих оснований математики на Coq.
На основе этого - наверняка можно строить одни и те-же структуры через разные леммы-теоремы максимизируя определенные тобой функционалы,и одостигать различные поставленные задачи.

Вряд ли придумывать позволит, но машинное хранение ипредставление под разным углом данных о теоремах - имхо перспективно, позволит(возможно) например избежать гигантских, неподъемных для других людей доказательств.

Я б этим бы занимался в качестве основной работы, да таких рабочих мест нет, а я не богатей - вот буду продолжать работать эникейщиком.
Аноним 19/05/16 Чтв 22:18:22  363895
>>363308
>>363532

проще некуда - вот представь, тебе надо разрезать сферу РОВНО пополам.

Ну потому что мы считаем неважным то, в какую сторону прямая проходит через начало координат.
(допустим из -oo в +oo или наоборот)


половинка без края (чашечка от бюстгалтера) слева, половинка без края - справа, а посередине еще не поделеннная окружность
ну делим её на две дуги окружности опять же без края, который представляет из себя две точки. Ну и, завершая, одну точку в первый набор, другую - во второй. Ура, поделили ровно пополам.

Теперь половину выбросили, остальное выпрямили, спроецировав как тангенс.
В итоге P2 = R^2+R+{.}

Аноним 19/05/16 Чтв 22:19:22  363896
>>363895
ну и сорян, я очень уставший(а когда неуставший,то здесь и не появляюсь)
Аноним 19/05/16 Чтв 22:34:26  363900
14636864664850.png (39Кб, 826x372)
Аноним 20/05/16 Птн 04:16:21  363912
14637069812370.jpg (4Кб, 227x83)
1. Пикрилейтед в списке простых задач для 11 класса - скорее всего опечатка, синус отдельно? Берётся ли вообще такой интеграл?
2. Будет ли иметь математический смысл уравнение, в котором есть все алгебро-тригонометрические действия, производные-интегралы, а также элементы групп и множеств?
Аноним 20/05/16 Птн 04:32:45  363913
>>363752
Лол, ну а тут сначала определяются свойства множеств, а потом топологическое пространство. Мне вот, если честно, все равно. То, что я буду знать что такое пространство мне понятие компактности не разъяснит. Я определение и через замкнутость с ограниченностью, и через конечное и бесконечное покрытие знаю, но пока я это не визуализирую - не пойму. Поэтому я и просил самое простое объяснение.
Аноним 20/05/16 Птн 04:46:46  363917
14637088067380.webm webm file (1970Кб, 1920x1080, 00:00:07)
Аноны, включаю я такой телик, а там Гриша Перельман идет в наушниках, я так и охуел. Вот он где оказывается. Фамилию поменял и в хоккей играет! Но окреп физически зато.
Аноним 20/05/16 Птн 04:48:32  363918
>>363917
Лол.
Аноним 20/05/16 Птн 05:22:15  363924
Поясните нубу, почему нельзя определить операцию сравнения для комплексных чисел.
Аноним 20/05/16 Птн 05:53:52  363926
>>363924
Потому что у действительных только одно направление на прямой по нему и сравнивают.
У комплексных два направления.
Аноним 20/05/16 Птн 06:06:40  363927
14637136001890.webm webm file (1562Кб, 384x288, 00:00:56)
14637136002001.webm webm file (3598Кб, 352x262, 00:00:52)
>>363900
Аноним 20/05/16 Птн 06:20:17  363928
>>363924
Сранивают же по модулю, не?
Аноним 20/05/16 Птн 06:25:26  363929
>>363928
Ещё и аргумент есть.
Аноним 20/05/16 Птн 07:48:22  363933
>>363926
Почему нельзя сравнивать по модулю?
Аноним 20/05/16 Птн 09:18:41  363940
>>363933
Можно, в принципе. Но тогда получается, что
1=i=-1 и 1+i=2, лол. 15+i2>16.
Аноним 20/05/16 Птн 09:35:54  363944
Есть ли полный список математических тем, начиная с теории множеств?
Аноним 20/05/16 Птн 09:38:45  363945
>>363944
Википедия?
Аноним 20/05/16 Птн 10:21:54  363947
>>363944
Есть MSC 2010.
http://rgho.st/6mdKrg4Cp
Аноним 20/05/16 Птн 13:33:51  363964
>>363912
1. Нет
2. Нет
Аноним 20/05/16 Птн 14:25:23  363971
>>363912
>Будет ли иметь математический смысл уравнение, в котором есть все алгебро-тригонометрические действия, производные-интегралы, а также элементы групп и множеств?

Так, так, так... Что тут у нас?..
Ага, очередная поебнятина!
Аноним 20/05/16 Птн 14:25:35  363972
14637435354820.png (69Кб, 1213x317)
>>363927
Аноним 20/05/16 Птн 14:29:00  363976
>Будет ли иметь математический смысл уравнение, в котором есть все алгебро-тригонометрические действия, производные-интегралы, а также элементы групп и множеств?
Да, почему бы и нет.
Аноним 20/05/16 Птн 14:29:01  363977
>>363940
Нет, я про сравнение между собой, а ты их с натуральными сравниваешь. Понятно, что при сравнении с натуральными получается дичь с точки зрения здравого смысла. Только i^2 = -1 тоже дичь с точки зрения здравого смысла. А если смотреть по графическому представлению комплексных чисел, то все довольно логично. 16 откладывается по одной оси, а 15 + i2 по двум. Гипотенуза больше суммы катетов, поэтому 15+i2 вполне может быть > 16. Не вижу принципиальной ошибки.
Аноним 20/05/16 Птн 14:33:43  363980
>>363940
Нахуй ты меня наебываешь? Я же тебе поверил.
15+i2<16, тащемто! Модуль 15+i2 это ~ 15.13 же.
Аноним 20/05/16 Птн 15:19:28  363986
>>363924
Отношение порядка для комплексных чисел определить можно, причём многими разными способами. Просто оно не будет согласована с арифметическими операциями. Есть две теоремы. 1: существует только одно линейно упорядоченное непрерывное поле, а именно вещественные числа. 2: поле комплексных чисел не изоморфно полю вещественных чисел. Эти теоремы означают, что комплексные числа нельзя сделать линейно упорядоченным непрерывным полем.
Аноним 20/05/16 Птн 17:31:34  364015
>1: существует только одно линейно упорядоченное непрерывное поле, а именно вещественные числа
Всё-таки полное по Дедекинду, а не "непрерывное".
Аноним 20/05/16 Птн 17:33:14  364017
>>364015
Синонимы.
Аноним 20/05/16 Птн 17:34:58  364019
>>364017
Какие-то необщепринятые синонимы:
https://www.google.com/webhp?sourceid=chrome-instant&ion=1&espv=2&ie=UTF-8#q=Continuous+ordered+field
с другой же стороны:
https://www.google.com/webhp?sourceid=chrome-instant&ion=1&espv=2&ie=UTF-8#q=dedekind+complete+ordered+field
Аноним 20/05/16 Птн 19:25:59  364038
Так все-таки, существует нормальная строгая книжка по дифурам? Пробовал Арнольда, но это пиздец, это же невозможно читать абсолютно.
Аноним 20/05/16 Птн 19:27:47  364039
>>364038
ramanan
ильяшенко-яковенко
Аноним 20/05/16 Птн 19:31:59  364040
>>364038
Эрроусмит
Аноним 20/05/16 Птн 19:45:32  364042
>>364039
> ramanan
Чет не гуглится. Как книга называется?
Аноним 20/05/16 Птн 19:47:10  364044
>>364042
Global calculus. Она в ОП-посте есть же.
Аноним 20/05/16 Птн 19:56:29  364046
>>364038
Kashiwara, Schapira, «Sheaves on Manifolds»
Аноним 20/05/16 Птн 21:24:58  364069
Помогите тупому
Докажите, что всякое бесконечное перечислимое множество можно представить в виде {a0,a1,…}{a0,a1,…} для некоторой вычислимой последовательности без повторений (ai≠ajai≠aj при i≠ji≠j). Можно ли усилить это утверждение, потребовав, чтобы слова были упорядочены по длине (длины слов ai не убывали с ростом i)?
Аноним 20/05/16 Птн 21:43:16  364073
В прошлой нити я спрашивал учебники про гридиенты и роторы, мне насоветовали пиндоский учебник. Сегодня я сдался его учить. Есть что то на русском? А то я без них нихуя максвеловские писульки не понимаю. Понимаю конечно, только не в том виде в котором их нынешние физеки написали
Аноним 20/05/16 Птн 21:46:55  364075
>>364073
Попробуй второй том Зорича.
Аноним 20/05/16 Птн 22:27:29  364087
14637724499570.gif (1690Кб, 500x500)
привет, я туповат. начал читать книгу по топологии, а нихуя не понимаю... то есть читаю, как автор книги раскрывает понятия, а мне не понятно, ни что он хочет сказать, ни как это себе представить. можно ли как-то изучать топологию, если уровень твоих знаний - слабо освоенная высшая математика в техническом вузе?
может кто-то на ютубе канал имеет с более понятным описанием понятий, или с лекциями.
прошу, анон, помоги, я уже почти отчаялся, хочу понять... всё...
Аноним 20/05/16 Птн 22:40:40  364092
>>364087
>можно ли как-то изучать топологию, если
Давай подойдём с другой стороны.

Почему ты решил изучать топологию? На кой ляд она тебе? Слово модное? Быть в тренде? Иное?
Аноним 20/05/16 Птн 22:41:54  364093
>>364075
А ты сам его читал хоть?
мимошёл
Аноним 20/05/16 Птн 22:42:52  364096
>>364093
Я прочитал полностью два тома Зорича и прорешал полностью первый и частично второй - это были первые два моих учебника по высшей математике, поэтому я немного предвзят - но мне они доставили дико.
Аноним 20/05/16 Птн 22:47:22  364097
14637736429960.png (102Кб, 1543x328)
>>364096
Я по-прежнему считаю, что Зорич не может в нотацию чуть более, чем совсем. Ну серьёзно, чем надо удолбиться, чтобы написать такое?
Аноним 20/05/16 Птн 22:48:36  364099
>>364097
"Если на минуту принять громоздкие символы", после чего он так нигде не пишет.
ЗОРИЧ Аноним 20/05/16 Птн 22:50:32  364100
Лох
Аноним 20/05/16 Птн 22:55:12  364104
>>364099
И плохо, что не пишет. В разуме Зорича разговор про окрестности почему-то прочно связан с этими вот упоротыми символами, и из-за этого он вообще не говорит про окрестности даже там, где про них говорить нужно.
Аноним 20/05/16 Птн 23:01:36  364106
14637744963440.png (391Кб, 1366x768)
>>364104
Не понимаю суть претензий. У него вполне милый и компактный язык предела по базе - на котором он и строит изложение анализа в своих учебниках - ровно как и нужно делать при концептуальном подходе.
Аноним 20/05/16 Птн 23:07:28  364107
>>363895
О, круто. Спасибо, анон.

>>363900
лол
Аноним 20/05/16 Птн 23:14:51  364109
14637752919710.jpg (4Кб, 217x241)
>>364046
Бля, ты че, серьёзно? Ну не трали, ну(
Аноним 20/05/16 Птн 23:16:05  364110
>>364096
Анон, а на каком ты сейчас курсе?
Аноним 20/05/16 Птн 23:17:58  364111
>>364046
А что не так? Ты ведь сам просил что-то пофундаментальнее. Можешь ещё навернуть Олвер "Группы ли и дифференциальные уравнения" (или как-то так), если чувствуешь, что не осилишь.
>>364110
На третьем.
Аноним 20/05/16 Птн 23:19:06  364113
>>364097
Да нормально всё. Вот Мочизука - там да, пиздец и обозначения, и новые нескучные понятия.
Аноним 20/05/16 Птн 23:37:15  364119
>>364092
Я не он, но я, например, изучаю топологию, чтобы понимать лекции Ромы Михайлова, потому что по нему видно, что он волшебник. Тоже хочу магию изучать. До алгебраической топологии мне еще далеко, но общую норм изучаю, теоремы доказываю, брат жив.
Аноним 20/05/16 Птн 23:42:36  364120
>>364087
The only prerequisite for general topology is some mathematical maturity.
Аноним 21/05/16 Суб 00:16:14  364127
>>364092
Я не он, но меня плющит от идеи, что тапалогии дают возможность изучать кучу вещей единообразными методами пользуясь универсальными понятиями типа непрерывностей, компактностей, связностей и прочей херни, а в алтопе так и вовсе обобщать обощения. Ну и Ромич, конечно, охуенен. Правда он на своей индусо-мистической волне, так что его авторитет и влияние тут скорее сбоку. Хотя воспринимать всю эту бороду, как некий язык мне тоже нравится. Но главным образом просто потому что это охуенно.
Аноним 21/05/16 Суб 00:23:27  364131
14637794073700.jpg (95Кб, 540x659)
>>364092
Меня вообще интересует вселенная, я читаю псевдонаучные книжки Хокинга, слабо соображаю во всем этом, но мне интересно. А тут я даже читать не могу, просто не понимаю
Аноним 21/05/16 Суб 00:25:33  364132
>>364119
>волшебник
Аноним 21/05/16 Суб 00:30:49  364135
>>364131
Попробуй переходный уровень. Возьми, например "теорию множеств" Вавилова - очень живо написано, но всё же это не перестаёт быть учебником.
Аноним 21/05/16 Суб 00:53:28  364142
>>364135
Ну ок. А за топологию вообще браться без шансов на понимание сейчас?
Аноним 21/05/16 Суб 00:55:35  364143
>>364142
Всё-таки нужна какая-то культура математических рассуждений, которая, всё же, нарабатывается на менее "продвинутых" разделах. Посиди там месяцок, а потом за топологию сможешь взяться.
Аноним 21/05/16 Суб 00:57:23  364144
14637814435500.jpg (126Кб, 720x721)
>>364143
Бля, ну ладно. Правда, мне сложно читать научную литературу с такой низкой мотивацией на результат, который придет (или нет) через месяц.
Но тут уже мои проблемы, спасибо за вопросы.
Аноним 21/05/16 Суб 01:47:40  364153
14637844602390.jpg (11Кб, 267x354)
>>364100
ТЫ ЧО СУКА ТЫ ЧО НАХУЙ
Только посмотри на это ехидное лицо. Даже ПЕКА-ФЕЙС смотрит на тебя с меньшем ехидством. В его взгляде чувствуется сарказм и презрение, тем кто не осилил его элементарный учебник.
Аноним 21/05/16 Суб 03:06:12  364155
>>364127
>изучать кучу вещей единообразными методами
Да это иллюзия же, "единообразие" кончается недалеко после определения топологического пространства.
Аноним 21/05/16 Суб 03:06:35  364156
>>364120
>mathematical maturity
Как формализовать?
Аноним 21/05/16 Суб 03:52:29  364157
>>364156
Формализация этого понятия равносильна созданию сильного искусственного интеллекта.
Аноним 21/05/16 Суб 04:30:08  364158
>>364144
редко я блокирую пикчи adblockом, ты выиграл приз моих симпатий
Аноним 21/05/16 Суб 12:37:35  364189
Репост из другого треда.
Не хочу создавать новый тред, поэтому спрошу тут. Нужна книга по ЯМРу, желательно от А до Я. Можно ингриш.
Аноним 21/05/16 Суб 13:08:52  364191
>>364156
Формализовать никак, а вообще вот
https://en.wikipedia.org/wiki/Mathematical_maturity
Аноним 21/05/16 Суб 13:14:07  364194
Поясните за форум http://mathhelpplanet.com/
там тоже старые совковые пердуны?
Аноним 21/05/16 Суб 13:19:23  364195
В ответе на вот этот вопрос
http://mathoverflow.net/questions/28721/good-differential-equations-text-for-undergraduates-who-want-to-become-pure-math
написано
> I've taught students who were gung-ho about rigorous real analysis, Rudin style, but couldn't compute the Taylor expansion of $\sqrt{1 + x^3}$.
Я тоже не смог. Я погуглил chain rule nth derivative и нашел длинную и уродливую Faa di Bruno's formula. Правда, что математик должен это уметь?

Аноним 21/05/16 Суб 13:52:32  364200
>>364195
Не только это, а еще и в сракотан долбиться.
Аноним 21/05/16 Суб 13:52:39  364201
>>364195
А, я просто тупой.
Нужно положить $y = x^3$ и разложить по формуле для $(1 + y)^\alpha$.
Аноним 21/05/16 Суб 13:53:18  364202
>>364201
Ну все, разложил, пойду дальше Рудина читать.
Аноним 21/05/16 Суб 14:28:37  364209
>>362618 (OP)
Бля, из всех предметов у меня проблемы только с дискретной математикой, мне кажется или там почти все аксиомы основываются не на фактах, а на "ятакскозал"(особенно теория графов вообще пушка)?

Посоветуйте какой-нибудь учебник для даунов с нормальными мат.выкладками, заранее спасибо.
Аноним 21/05/16 Суб 15:29:05  364218
14638337458900.jpg (33Кб, 500x368)
Здравствуйте.
Я пишу статью.
Извольте объективно раскритиковать или же обратное, если будет угодно.
http://lvlb.ru/Void/Continuum.html
Аноним 21/05/16 Суб 15:42:20  364222
>>364218
0 не натуральное число.Еще проще будет, когда записывать числа в одиничной системе(1) вот тебе "еще проще".
Аноним 21/05/16 Суб 15:44:49  364224
>>364222
натуральное число представимо в бинарном виде
01
10
11
100
101 etc
речь об этом (см. диагональную конструкцию кантора)
да, через простые объекты, пусть будет пустота - можно любое число записать, спасибо кстати есть над чем подумать тут
Аноним 21/05/16 Суб 15:44:52  364225
>>364222
0 - натуральное число.
Аноним 21/05/16 Суб 15:48:13  364226
>>364224
Прекрасный метод! Предлгагаю доказательство гипотезы Римана:
Понимание - это сведение одного типа реальности к другому Леви-Стросс
Кто ясно мыслит, ясно излагает Никола Буало
О чём нельзя говорить, о том лучше молчать Витгенштейн

Всё просто
Любое натуральное число представимо в двоичной системе счисления.
Теорема доказана. Гипотеза Римана верна.

Аноним 21/05/16 Суб 15:50:31  364227
>>364226
По существу критика есть?
Метаматематика выходит за пределы математике, здесь мы обсуждаем континуум-гипотезу, недоказуемую и неопровержимую в ZFC.
Вы ждали другого решения?
Аноним 21/05/16 Суб 15:53:18  364228
>>364227
По существу критика: этот словесный пассаж ни к математике ни к (тем более) метаматематике отношения не имеет.
Аноним 21/05/16 Суб 15:56:30  364231
>>364227
В zfc всего два аксиоматических утверждения:множество и принадлежность, все остальное строится на них.Логически они друг другу никак не противоречат, потому что за парадоксы вы тут раскапываете, я так и не понял.
Аноним 21/05/16 Суб 15:57:05  364232
>>364231
Справедливости ради - ты тоже сказал лютейшую хуйню.
Аноним 21/05/16 Суб 15:58:00  364233
14638354802040.jpg (28Кб, 659x249)
>>364228
Советую тебе сохранить свой пост, дабы узреть его истинность в будущем.
Аноним 21/05/16 Суб 16:53:53  364237
>>364218
Поверю, только если переделаешь на сайте Бога
Аноним 21/05/16 Суб 17:05:25  364238
>>364237
я не Бог, мне все равно
Аноним 21/05/16 Суб 18:07:13  364248
>>364218
если писос мой пососешь, я признаю доказательство верным

мочизука
Аноним 21/05/16 Суб 18:10:31  364250
>>364231
бурбаки вы везде блять
Аноним 22/05/16 Вск 05:02:52  364368
>>364225
>0 - натуральное число.
Бурбаки в треде, все в Арнольда.
Аноним 22/05/16 Вск 06:31:20  364373
почему так давно не было мат срачей на пицот постов? почему никто не определяет N и не доказывает что pi=4? что происходит? все стали учебники читать по списку и стесняются теперь хуйню пороть что ли? двач это теперь место толковых дискуссий, так получается?
Аноним 22/05/16 Вск 08:21:10  364380
>>364373
ЕГЭ же. Школоте не до срачей. Подожди лета.
Аноним 22/05/16 Вск 08:55:17  364383
>>364368
Это общепринятая точка зрения. Объекты, о которых идёт речь, - это ординалы, меньшие омега (они же кардиналы, меньшие алеф-нулевого). Ординал пустого множества, конечно, меньше омеги. Поэтому ординал пустого множества входит в обсуждаемую совокупность объектов.
Аноним 22/05/16 Вск 10:18:47  364395
>>364383
Ты не понял отсылку на войну Арнольда с Бурбакизмом.
Аноним 22/05/16 Вск 10:29:59  364397
>>364395
Понял, разумеется. Просто посчитал нужным сказать, что Бурбаки тут ни при чём.
Аноним 22/05/16 Вск 10:32:28  364398
>>364397
>Бурбаки тут ни при чём
http://www.mccme.ru/conf2000/tez_arn.htm
>Обучение математике, как своеобразной юридической казуистике, основанной на произвольно выбранных законах, начинается с самого раннего возраста: французских школьников учат, что любое вещественное число больше самого себя, что 0 --- натуральное число, что всё общее и абстрактное важнее частного, конкретного.
Аноним 22/05/16 Вск 10:42:55  364400
>>364398
И чо? Не Бурбаки придумали, что 0 - натуральное число. И это не произвольно выбранный закон, это разумное предложение. Зато у борцунов с натуральным нулём никаких аргументов нет. С какой стати натуральные числа должны начинаться с 1?
Аноним 22/05/16 Вск 11:12:25  364412
То, что 0∈N действительно выглядит разумно, но нумерация с 0 раздражает. Нумерация начинается с 1, объект номер ноль, блеать.
Аноним 22/05/16 Вск 11:14:40  364413
>>364412
В чём проблема? Номер - это смещение от начала. Ясно, что у начального объекта нулевое смещение.
Аноним 22/05/16 Вск 13:18:49  364481
>>364373
Моча всё стирает оперативно.
Аноним 22/05/16 Вск 14:14:27  364507
>>364412
Перед тем, как спорить о том 0 натуральное ли число. Надо определить N. В аксиомах пеано нет нуля.
Аноним 22/05/16 Вск 14:27:47  364510
>>364507
Что в словах "конечные ординалы" тебе непонятно?
Аноним 22/05/16 Вск 15:32:38  364529
14639203588520.webm webm file (4846Кб, 480x360, 00:01:23)
>>364119
>чтобы понимать лекции Ромы Михайлова, потому что по нему видно, что он волшебник. Тоже хочу магию изучать.
Тогда тебе скорее вот это читать нужно:
https://storage.googleapis.com/content-krot-me/ravinagar/ravinagar.pdf
https://www.youtube.com/watch?v=QAD9c-9yHsc

И прочую шизотерику.
Аноним 22/05/16 Вск 15:35:30  364530
>>364529
Уже прочитал. И все видосы, где он шарами жонглирует и танцует посмотрел. Но я хочу алгебраическую топологию.
Аноним 22/05/16 Вск 15:47:59  364536
>>364510
при чем тут ординалы, мы говорили об аксиомах пеано.
Аноним 22/05/16 Вск 16:05:28  364549
14639223286290.webm webm file (3495Кб, 480x360, 00:00:42)
>>364530
>И все видосы, где он шарами жонглирует и танцует посмотрел.
Обрёл просветление?
Аноним 22/05/16 Вск 16:11:50  364552
>>364536
Определение. Натуральные числа - конечные ординалы.

Всё, иди еби гусей.
Аноним 22/05/16 Вск 16:22:06  364553
>>364552
>Натуральные числа - конечные ординалы
Докажи это. Не зря натуральные числа с единицы начинаются.
Аноним 22/05/16 Вск 16:22:59  364554
>>364412
>но нумерация с 0 раздражает
Сразу видно, что ты не программист.
Аноним 22/05/16 Вск 16:25:06  364555
>>364553
Определение. Натуральные числа - конечные ординалы.
Теорема. Натуральные числа - конечные ординалы.
Доказательство. По определению.
Аноним 22/05/16 Вск 16:39:39  364561
>>364555
Определение. Натуральные числа начинаются с единицы.
0 меньше 1. 0 не натуральное число.
Доказательство. По определению.
Аноним 22/05/16 Вск 16:40:01  364562
>>364561
Лол, слив засчитан. Иди нахуй.
Аноним 22/05/16 Вск 16:41:15  364563
>>364562
Ты ведь понимаешь, что таким образом сам себя сливаешь?
Аноним 22/05/16 Вск 16:43:11  364567
>>364563
Иди-иди.
Аноним 22/05/16 Вск 16:45:29  364569
14639247296450.png (52Кб, 1042x451)
>>364567
Иди к ЕГЭ готовится, школьник.
Аноним 22/05/16 Вск 16:46:41  364570
>>364569
Иди нахуй.
Аноним 22/05/16 Вск 16:47:27  364571
>>364570
Мамку свою посылать будешь.
Аноним 22/05/16 Вск 16:52:10  364576
>>364569
Википедия - это не место для спецификации определений и теорем - в учебниках аксиомы пеано встречаются как с единицы, так и с нуля. Не говоря уже о том, что с точки зрения общей теории - это канонически изоморфные системы.
Аноним 22/05/16 Вск 16:53:56  364577
>>364569
И в английской википедии начинается таки с нуля https://en.wikipedia.org/wiki/Peano_axioms#Formulation
Аноним 22/05/16 Вск 17:05:25  364578
>>364577
Бездуховные пендосы.
Аноним 22/05/16 Вск 17:12:08  364580
>>364577
>Peano's original formulation of the axioms used 1 instead of 0 as the "first" natural number.
Аноним 22/05/16 Вск 17:12:52  364581
>>364580
Кого ебёт историзм в математике?
Аноним 22/05/16 Вск 17:15:21  364582
>>364581
Кого ебёт бурбакизм в математике?
Аноним 22/05/16 Вск 17:15:50  364583
>>364582
При чём тут Бурбаки?
Аноним 22/05/16 Вск 17:16:46  364584
>>364583
ссылаюсь на арнольда
Аноним 22/05/16 Вск 17:17:03  364585
>>364582
Всех ебёт, на самом деле.
Аноним 22/05/16 Вск 17:26:47  364587
>>364584
Отъебись.
Арнольд
Аноним 22/05/16 Вск 17:27:08  364588
>>364585
Арнольда уже выебали.
Аноним 22/05/16 Вск 18:20:06  364594
"Математики, кажется, сходятся на том, что между нашими "интуитивными" представлениями о множествах и числах и призванными их описывать формализмами имеется не более чем поверхностное сходство. Разногласия относятся лишь к вопросу о выборе между теми и другими." Н.Бурбаки
Аноним 22/05/16 Вск 18:30:48  364595
>>364549
Ну не все, конечно, понятно, но в целом мироощущение у пацана правильное, у меня похожее.
Аноним 22/05/16 Вск 18:39:17  364597
14639315570060.webm webm file (786Кб, 1280x720, 00:00:07)
>>364398
>французских школьников учат, что любое вещественное число больше самого себя, что 0 --- натуральное число, что всё общее и абстрактное важнее частного, конкретного.
Что не так? Ну, кроме того что Арнольд видимо обосрался и под
>что любое вещественное число больше самого себя
имел ввиду отношение порядка ≥, которое определяется до >.
Конечно не с этого нужно школьникам начинать, но со старших мат.классов нужно перекатываться во "всё общее и абстрактное важнее частного, конкретного."
Аноним 22/05/16 Вск 19:04:40  364599
>>364594
Всё правильно написали.
Аноним 22/05/16 Вск 19:17:42  364601
Добавьте высшую алгебру для самых маленьких (для первого знакомства):

- A Book of Abstract Algebra by Charles C Pinter (пожалуй, лучший элементарный учебник по высшей алгебре, который можно давать даже школьникам средних классов - и это не преувеличение, мол он так хорошо написан, это реальность)

- Survey of Modern Algebra by Garrett Birkhoff and Saunders MacLane (не путать с просто "Algebra" от тех же авторов, который будет посложнее и отображает категориальный подход к алгебре: МакЛейн - один из создателей теории категорий)
Аноним 22/05/16 Вск 19:24:45  364602
>>364218
>>364226
P = NP когда N = 1 или P = 0
/proof
Аноним 22/05/16 Вск 19:32:49  364604
>>364602
Охуенная шутка, новая, наверное.
Аноним 22/05/16 Вск 19:39:41  364605
Посоветуйте книгу-быстрое ревью элементарной школьной геометрии (до 100 страниц). Можно на английском.
Аноним 22/05/16 Вск 19:52:07  364606
>>364605
Шень "Геометрия в задачах", за вычетом задач где-то 50-70 страниц и будет
Аноним 22/05/16 Вск 20:46:26  364610
Если кто хочет самостоятельно подрочить упражнения в анализе, но не уверен в своих силах и правильности своих доказательств, вот есть такой двухтомник (на либгене есть):
http://www.amazon.com/Exercises-Analysis-Problem-Books-Mathematics/dp/3319061755/ref=sr_1_2?s=books&ie=UTF8&qid=1463939093&sr=1-2&keywords=Exercises+in++Analysis

http://www.amazon.com/Exercises-Analysis-Nonlinear-Problem-Mathematics/dp/3319278150/ref=sr_1_1?s=books&ie=UTF8&qid=1463939093&sr=1-1&keywords=Exercises+in++Analysis
Аноним 22/05/16 Вск 21:09:21  364612
14639405617320.jpg (32Кб, 605x305)
>>362618 (OP)
Аноны, хэлп. Поделитесь своими мыслями. Жизнь решается.
Аноним 22/05/16 Вск 21:13:13  364614
>>364612
Задачи сложные, на самом деле, где учишься?
Аноним 22/05/16 Вск 21:14:21  364616
>>364614
Мехмат МГУ, первый курс. Прошу помощи т.к. еще много дел нужно решить по другим дисциплинам.
Аноним 22/05/16 Вск 21:15:31  364617
>>364616
Есть время или тебе надо в ближайшие часы сделать?
Аноним 22/05/16 Вск 21:17:28  364618
>>364617
В среду сдача задачек.
Аноним 22/05/16 Вск 21:18:34  364619
>>364618
Ничего не обещаю, но попробую что-то решить.
Аноним 22/05/16 Вск 21:19:37  364621
>>364619
Если что-то получится, буду премного благодарен.
Аноним 22/05/16 Вск 21:23:27  364623
>>364612
http://www.wolframalpha.com
Аноним 23/05/16 Пнд 01:39:19  364644
>>364561
Но ведь в наше время плохо живется без нейтрального элемента. Или тебе норм?
Аноним 23/05/16 Пнд 13:45:53  364693
>>364644
Если долбится в сракотан как он то норм.
Аноним 23/05/16 Пнд 16:50:08  364725
14640114088780.png (60Кб, 400x388)
>>362618 (OP)
реквестирую годный сервис генерации математический задач с начального класса по последний курс с возможностью кастомизации задач
Аноним 23/05/16 Пнд 18:53:23  364750
>>364529
Ромка-то еще и артист, получается.
Аноним 23/05/16 Пнд 19:06:15  364751
>>364750
А то как же.
https://www.youtube.com/watch?v=DLvpcsXz-1c
Аноним 23/05/16 Пнд 19:58:10  364771
>>364751
Ну iбанiй в рiт!
Аноним 23/05/16 Пнд 20:02:35  364772
>>364725
Сгенерированные задачи годными быть не могут.
Аноним 24/05/16 Втр 15:07:00  364929
>>362636
Встаньте на раздачу кто-нибудь,пожайлуста
Аноним 24/05/16 Втр 17:08:47  364945
сделал djvu bookmarks для Винберг - курс алгебры
https://gist.github.com/anonymous/4128c4679588b3d81f77dfe946f6ccf3

могу djvu выложить, но не знаю куда.
Аноним 24/05/16 Втр 17:14:01  364948
Предыдущие треды архивировались? В шапке нет, а через архивач находятся только #1 #2 #3 или руки кривые. Как вообще архивация происходит, автоматически?
Аноним 24/05/16 Втр 17:18:32  364951
>>364929
сиды плз поднажмите!!!!!!
Аноним 24/05/16 Втр 18:45:47  364964
>>364948
Нет, нужно вручную добавлять на архивач.
Аноним 24/05/16 Втр 20:49:26  364998
>>364549
Эти типы с Ромчиком, как-то пристали ко мне, когда я был малым. Я даже не знал, что значат все эти слова.

Вышел забавный разговор:
- Что у них там, капитализм? А у нас социализм!
- Капитализм лучше!
- А у нас будет всё вместе!
- Та постой, смотри прикол!..
- Сейчас с РПГ вас расхуярю.
- А где возьмёшь?
- Вот этот рандомный тип, поднесёт.
(Аххаххах, малый устроит экзекуцию!...)
Аноним 24/05/16 Втр 20:51:28  365000
>>364929
Я обычно ночью стою, сори.
Аноним 25/05/16 Срд 14:24:46  365109
>>364595
>правильное
классический уход от реальности, очень распространен в рашке. каких только философов не рождала эта страна.
Аноним 25/05/16 Срд 15:51:22  365118
>>365109
Иногда реальность заслуживает того, чтобы от неё уйти.
Аноним 25/05/16 Срд 17:02:20  365121
>>365118
В математику хуево уходить от реальности. Даже попиздеть будет не с кем. Лучше в диванную философию конечно.
Аноним 25/05/16 Срд 17:03:46  365124
>>365121
В философии нет никакой объективности, любые два текста равноценны. А в математике есть определенность.
Аноним 25/05/16 Срд 17:48:49  365131
>>365124
>А в математике есть определенность.
Да, например если ты ультрафинитист или интуиционист - то принято покрывать тебя хуями, а если ты АЛГЕБРАИЧЕСКОЙ ГЕОМЕТРИЕЙ занимаешься - то ты хороший, годный математик.
Аноним 25/05/16 Срд 17:56:01  365132
>>365131
Что ультрафинитист, что алгеометр пользуются одним и тем же методом - пользуясь логикой, выводят следствия из определений. набор аксиом - тоже определение. Разница между ними просто в наборе используемых определений.

А в философии вообще метода нет.
Аноним 25/05/16 Срд 17:57:42  365134
>выводят следствия из определений
>выводят следствия из определений
Ясно.
>А в философии вообще метода нет.
Как ни странно, метод точно такой же (с точностью до поправок).
Аноним 25/05/16 Срд 17:58:55  365135
>>365134
Да не, фейлософия - это не то.
Аноним 25/05/16 Срд 18:04:04  365136
>>365135
Много великих физиков и математиков занимались философией.
Аноним 25/05/16 Срд 18:35:23  365141
>>365134
Именно так. Следствия выводятся из определений. Всякое определение - перечень аксиом, всякий перечень аксиом - определение.

А в философии нет ни определений в математическом смысле, ни даже просто фиксированной логики. Философы образуют свои тексты без всяких правил, без системы, без порядка.
Аноним 25/05/16 Срд 20:07:05  365158
>>365141
>Следствия выводятся из определений.
Ясно, мат.логику в школе не проходят.
>А в философии нет ни определений в математическом смысле, ни даже просто фиксированной логики. Философы образуют свои тексты без всяких правил, без системы, без порядка.
Лел, ну у математиков-то логика уж точно фиксирована. Особенно когда читаешь статью по гомотопической топологии и по теории вероятностей - рассуждения просто один в один.
Аноним 25/05/16 Срд 20:55:07  365180
Я попробовал почитать второй том зорича чтобы выучить статоры с роторами, но он какими то загадками говорит. Я посмотрел первый том, там такая же хуйня. То что можно было объяснить двумя картинками и кончиком пальца он запихивает в какие то логические цепочки которые без дешифратора читать невозможно.
Дайте другой учебник по роторам и градиентам.
Аноним 25/05/16 Срд 20:58:12  365184
Как, в общем-то, доказывать что-либо? Просто нужно более-менее понимать уже имеющиеся и потом, с набором опыта, придёт само? Или надо сидеть над доказательными задачами, ожидая просветления? Или что-то совсем среднее?

>>365141
>Следствия выводятся из определений
Следствия исходят из теорем, основанных на определениях и аксиомах. И на других теоремах, разумеется.
Аноним 25/05/16 Срд 21:01:05  365186
>>365184
Хуй там. Аксиома и определение принимаются за базу. Маняматика с её пифагоровыми штанами и зфц теориями стоит на том что два камня плюс два камня будет четыре камня, а две непересающиеся прямые парралельны.
Аноним 25/05/16 Срд 21:08:14  365189
>>365180
Ты просто неосилятор.
Аноним 25/05/16 Срд 21:10:32  365191
>>365186
Выучи сначала, что такое определение и что такое аксиома, а потом рассуждай, иначе поступаешь точно так же, как и ненавидимые тобою "феласафы" (или, по крайней мере, их образ в твоей фантазии).
Аноним 25/05/16 Срд 21:17:06  365194
>>365158
>Ясно, мат.логику в школе не проходят.
Ты хочешь что-то возразить?
Аноним 25/05/16 Срд 21:19:57  365197
>>365194
Хочу. Из определений не выводятся утверждения. Утверждения выводятся из (других или тех же самых) утверждений.
Аноним 25/05/16 Срд 21:36:42  365207
>>365197
Аксиома - это утверждение. Определение - это перечень аксиом.
Аноним 25/05/16 Срд 23:22:08  365251
>>365191
Ты с разными людьми говоришь.
Аноним 26/05/16 Чтв 00:14:45  365258
>>365207
Первое верно, но это не определение. Второе неверно.
Аноним 26/05/16 Чтв 00:32:53  365260
>>365258
Определение это же просто верное предложение, не?
Аноним 26/05/16 Чтв 00:43:04  365261
>>365258
Второе, разумеется, верно. Определение не есть что-то большее, чем просто несколько аксиом.
Аноним 26/05/16 Чтв 00:48:09  365262
>>365261
https://en.wikipedia.org/wiki/Extension_by_definitions
Аноним 26/05/16 Чтв 00:50:48  365263
>>365262
So?
Аноним 26/05/16 Чтв 00:53:11  365264
>>365263
Если ты не видишь разницы между консервативным расширением теории и аксиомой - это твои проблемы.
"Из определения выводятся теоремы" - долбоебизм и с точки зрения семантики русского языка и с точки зрения формальной математики.
Аноним 26/05/16 Чтв 00:55:56  365265
>>365264
Статья, ссылку на которую ты дал, никак не подтверждает твою правоту. Сформулируй свою точку зрения внятно. Что такое определение, по-твоему?
Аноним 26/05/16 Чтв 01:03:35  365266
>>365265
Ну если ты не умеешь читать, то окей.
Определение реляционного символа R(t1,t2,...,tn) в теории первого порядка Т. это тройка (T',R,phi_R) где T' - это консервативное расширение теории Т, R(t1,t2,...,tn) - реляционный символ порядка n, phi_R(t1,t2,t3,...,tn) формула первого порядка со свободными переменными t1,t2,...,tn.
T' содержит точно такие же правила вывода, аксиомы, реляционные и функциональные символы что и теория Т, за исключением одного нового реляционного символа R(t1,t2,...,tn) и одной новой аксиомы forall t1,t2,t3,...,tn(R(t1,t2,...,tn) <-> phi_R(t1,t2,t3,...,tn)).
Аноним 26/05/16 Чтв 01:10:12  365267
>>365266
Ну так и что с того? Ты вообще понимаешь смысл слов, которые пишешь? Я с тобой об определениях говорю, анон. Определения - это тексты на метаязыке. Чтобы вводить определения, вообще не обязательно вводить в теорию новый символ.
Аноним 26/05/16 Чтв 01:18:33  365268
В чём ценность философии?
Аноним 26/05/16 Чтв 01:19:19  365269
>>365267
Так метаязык полностью интерпретируется в неметаязыке, в этом ведь и вся фишка. Впрочем, тоже самое можно проинтерпретировать и на русском. Определение - это введение некоторого значка, который является сокращением для некоторой формулы. Не знаю, как из сокращения для формулы можно что-то вывести.

Давай конкретный пример, вот есть понятие = "принадлежит" выведи мне что-нибудь из него.

>>365268
В том же самом, в чём и у фундаментальной математики.
Аноним 26/05/16 Чтв 01:25:54  365270
>>365269
Определение - это совокупность Г аксиом, которая может, но не обязана, иметь метаязыковое имя. Мне нужно объяснить тебе, что такое синтаксическое следствие из множества формул?
Аноним 26/05/16 Чтв 01:28:33  365271
>>365270
>Определение - это совокупность Г аксиом, которая может, но не обязана, иметь метаязыковое имя.
Пруф в виде хотя бы одного учебника, в котором такое было бы написано.
Аноним 26/05/16 Чтв 01:32:00  365272
>>365271
Я только что определил, что такое определение. Какой ещё пруф тебе нужен?
Аноним 26/05/16 Чтв 01:32:34  365273
>>365272
Твоя терминология различается с общепринятой.
Аноним 26/05/16 Чтв 01:35:17  365274
>>365273
Я не вижу ничего плохого в этом. Теперь, когда мы прояснили этот вопрос, у тебя остались ко мне претензии?
Аноним 26/05/16 Чтв 01:43:23  365275
>>365272
А твое определение определения является по себе определением?
Аноним 26/05/16 Чтв 01:48:57  365276
>>365274
Нет.
Аноним 26/05/16 Чтв 02:25:19  365277
>>365272
А бывают пруфы для определений? Например определение понятия действия. Или определение числа. Дал определение, а теперь докаже что оно верно. Или например пруф самого понятия "определение". Если у вас есть ответ на вопрос, что такое определение, можно ли доказать, что ваш ответ - правильный?
Аноним 26/05/16 Чтв 02:35:22  365278
>>365277
>А бывают пруфы для определений?
Нет.
Аноним 26/05/16 Чтв 08:54:25  365284
>>362618 (OP)
Блядь, чё за гавно в "ДЛЯ САМЫХ МАЛЕНЬКИХ"?
Кароче, берете
>(1)И. М. Гельфанд, А. Шень: “Алгебра”
>(2)И. М. Гельфанд, С.М. Львовский, А. Л. Тоом: “Тригонометрия”
>(3)А.Шень "Геометрия"
>(4)Б. Давидович, П. Пушкарь, Ю. Чеканов "Матан для 5семитов"
>(5)В. Алексеев "Теория Абеля"
Откладываем (5) и (4). Налегаете сначала на (1) попутно решая (3). Не обязательно читать её от корки до корки(но большую часть нужно) и прорешивать все задачи(но большинство нужно; если название темы вам знакомо - пробегаетесь глазами по параграфу и приступаем к задачам. Встретили трудность, то прочтите параграф внимательней)
Закончив с (1) и что-то прорешав в (3) открываем (5) и (2) ((3)продолжаем решать). К концу I главы (5) вы должны прочесть(и решить задачи) (2) до последнего параграфа(комплексные числа).
Начав II главу в (5) приступаем к (4). Решаем до конца.

Не налегайте на (3) слишком сильно, несколько задач в день должно вам хватить, чтобы отвлечься от основного.
Аноним 26/05/16 Чтв 08:57:56  365285
>>365284
Ещё можно добавить
>А. Шень "начала теории множеств"
Аноним 26/05/16 Чтв 08:59:41  365286
>>365284
>В. Алексеев "Теорема Абеля"
быстрофикс
Аноним 26/05/16 Чтв 11:26:10  365294
>>365284
Не обращай внимания. Тут сплошь дауны-бурбакисты. Я уже пытался два треда подряд сделать нормальный список, но потом просто смирился с котуженными теорией множеств бурбакистами.
Аноним 26/05/16 Чтв 11:32:07  365295
>>365284
>Шень Геометрия
Не курс ли это ненужной планиметрии?
Аноним 26/05/16 Чтв 11:33:22  365296
>>365285
Невероятно унылый и бесполезный учебник. Стоит навернуть только любителем оснований математики.
Аноним 26/05/16 Чтв 11:50:41  365298
>>365295
Ладна, давай ты мне пояснишь, почему она не нужна, учитывая:
>ДЛЯ САМЫХ МАЛЕНЬКИХ
Аноним 26/05/16 Чтв 12:16:20  365300
>>365296
>основаниря математики
>теория множеств
В еще толще можешь?
Аноним 26/05/16 Чтв 14:58:00  365310
>>365300
А кому еще нужна ТМ в таком объеме?

>>365298
Ну, подразумевая, что ЦА данного поста хотя бы старшеклассники, они уже наелись планиметрии в школе, зачем еще то? Она ведь не является необходимым шагом, просто развивает умение писать доказательства и воображение.
Аноним 26/05/16 Чтв 16:10:44  365314
Да вы охуели (на самом деле, спасибо всем)! Как мне всю накиданную литературу в список компилить?
Аноним 26/05/16 Чтв 16:24:02  365317
>>365314
Сперва прочитать. Потом выкинуть никому нахуй не нужную, кроме трех с половиной обрыганов, теорию множеств. Потом бурбакодаунство. Потом уже компилировать по уровням сложности.
Аноним 26/05/16 Чтв 16:28:08  365319
>>365317
Ничего выкидывать не надо. Задача этого списка - быть максимально разнообразным.
Аноним 26/05/16 Чтв 17:03:01  365322
>>365260
>>365272
Но как проверить, что определение определения верно? Где граница между набором слов и математикой?
Аноним 26/05/16 Чтв 17:07:20  365323
По ТМ читать Вавилова - охуенен.
Аноним 26/05/16 Чтв 17:10:41  365324
Аноны, вот есть лингвисты - они на языках нихуя не говорят, я читал когда-то этих долбоебов в soc.lang или какая у них там ньюсгруппа. Они хуесосят друг друга похлеще чем здесь, но мне всегда смешно, что в сами языки они не могут почти, зато знают до хуя ПРО языки. Мне кажется здесь тоже часть бесед в таком же ключе. Не как что-то плохое, just saying.
Аноним 26/05/16 Чтв 17:11:38  365325
>>365324
Так и есть ^^
Аноним 26/05/16 Чтв 17:12:22  365326
>>365324
Нахуй пошёл.
Аноним 26/05/16 Чтв 17:15:09  365327
14642721099650.png (22Кб, 1219x151)
>>365324
Некоторые люди здесь кое-что могут. Например, могут решить задачку пикрелейтед.
Аноним 26/05/16 Чтв 17:15:58  365328
>>365327
Да, это очевидно.
Аноним 26/05/16 Чтв 17:17:26  365329
>>365328
Чому? Они же не пересекающиеся. А значит, если их не бесконечно много, то на прямой может найтись отрезок, где белого цвета нет вообще.
Аноним 26/05/16 Чтв 17:18:49  365330
>>365329
На шаге n напихаем на прямую столько отрезков. чтобы не существовало чёрного отрезка длины больше 1/n. Сделаем счётное число шагов.
Аноним 26/05/16 Чтв 17:20:26  365331
>>365327
>>365326
Сорри братья, я не имел ввиду что вы ничего не можете, вы охуенны.
Аноним 26/05/16 Чтв 17:21:22  365332
Я заебался. Сейчас мне приходиться читать мат. литературу университетского уровня на английском и другого выбора нет, но, так как мои познания в этом языке максимум на уровне восьмиклассника, как же это сложно, блядь! Каждое пятое встреченное английское слово - незнакомое, приходиться лезть в ебучий транслейт, смотреть ебучее слово, понимать, что эта хуйня значит в контексте от ебучего автора! Блядь! И так каждые тридцать секунд!
Половина моих сил уходит не на понимание математики, а на заучивание слов языка. Что посоветуете делать? Дальше ебаться с такой хуйнёй и таким образом выучить язык со временем или экстренно восполнить катастрофически нужный английский словарный запас? Есть какие-нибудь интернет-ресурсы?
Аноним 26/05/16 Чтв 17:22:29  365333
>>365332
А мне кажется наоборот, грамматика простейшая, фокус только на лексике. Слова тоже часто угадываются. Кроме тех, что хуй догадаешься.
Аноним 26/05/16 Чтв 17:22:42  365334
> Дальше ебаться с такой хуйнёй и таким образом выучить язык со временем
This, в книгах по математике 2-3 тысячи слов, ёбта, привыкнешь.
Аноним 26/05/16 Чтв 17:22:49  365335
>>365329
Рассмотрим множество всех семейств отрезков, в которых отрезки покрашены в белый цвет и попарно не пересекаются.

Частично упорядочим это множество по отношению "быть подсемейством". Ясно, что объединение любой цепи в этом множестве - снова семейство попарно непересекающихся отрезков, покрашенных в белый цвет.

Таким образом, любая цепь мажорируется. По лемме Цорна, существует максимальная цепь T.

Предположим, что на каком-то отрезке нет белых точек. Тогда его можно покрасить в белый цвет и добавить к T - что противоречит максимальности T.
Аноним 26/05/16 Чтв 17:28:52  365337
>>365332
Блядь, а один раз долбоебучий автор ебучего учебника решил пошутить про то, что математики не любят рыбу. А так как шутки в принципе переводятся сложнее, с этой хуйнёй мучался минут пять. 5 МИНУТ, БЛЯДЬ! Какая-то рыба в мат. тексте, что я мог ещё делать? Только потом понял, что автор долбаёб и шутит. Да если бы я знал с самого сначала, что это шутка, я бы к этому учебнику вообще больше не притронулся.
Аноним 26/05/16 Чтв 17:30:11  365338
>>365334
Эх, ладно. Излил вам душу, хоть возгорание потушил.
Продолжу ебаться с хуйнёй.
Аноним 26/05/16 Чтв 17:30:30  365339
>>365330
Так это только половина пруфа. Теперь докажи, что это напихание можно осуществить.
Аноним 26/05/16 Чтв 17:31:16  365340
>>365334
и из них только 30% мат. терминов, а остальное common words. и примерно 70% этих терминов будут и во второй книге по гамологиям. так что
говно вопрос.

>>365337
в чем шутка про рыбу? so long and thanks for all the fish?
Аноним 26/05/16 Чтв 17:49:10  365347
>>365334
Ну тогда можно начать с геометрии берже
Аноним 26/05/16 Чтв 17:56:03  365350
>>365335
> Ясно, что объединение любой цепи в этом множестве - снова семейство попарно непересекающихся отрезков, покрашенных в белый цвет.
Мне неясно.
Аноним 26/05/16 Чтв 17:59:58  365352
>>365350
ясно/очевидно что - это стандартная конструкция в математике? или нужно говорить "по определению" из чего ясно.
Аноним 26/05/16 Чтв 18:06:27  365353
>>365350
Пусть X1, X2, X3, ... - цепь из семейств попарно непересекающихся белых отрезков, т.е. X1 ⊂ X2 ⊂ X3 ⊂ ...

Пусть семейство белых отрезков X - объединение этой цепи. Предположим, что a и b - пересекающиеся отрезки из X. Тогда существуют такие числа m и n, что a - элемент семейства Xm, b - элемент семейства Xn. Пусть k = max(m,n). Тогда и a, и b - элементы Xk и потому не пересекаются.
Аноним 26/05/16 Чтв 18:13:29  365355
курлы
Аноним 26/05/16 Чтв 18:15:16  365358
Перекотейка

>>365356 (OP)
Аноним 26/05/16 Чтв 20:00:25  365383
>>365329
Платина?
Аноним 26/05/16 Чтв 20:00:59  365384
>>365329
Платина?
Аноним 27/05/16 Птн 01:17:19  365433
>>365339
Это очевидно.
Аноним 03/06/16 Птн 22:08:39  367135
test
Аноним 13/09/16 Втр 05:32:45  387166
>>362618 (OP)
шапку поправьте

[Назад][Обновить тред][Вверх][Каталог] [Реквест разбана] [Подписаться на тред] [ ] 536 | 44 | 145
Назад Вверх Каталог Обновить

Топ тредов
Избранное